Vous êtes sur la page 1sur 68

INSURANCE LAW

I. Introduction

a. laws governing insurance

1. Enriquez vs. Sun Life of Canada, GR L-15895, November 29, 1920

FACTS:

Joaquin Herrer made an application with Sun Life for a life annuity. He paid the amount of
P6,000.00 to the Manila manager who gave him a "provisional" receipt "subject to medical
examination and approval of the Company's Central Office." The application was forwarded to
the head office in Canada and the policy was issued on December 4, 1917 in Canada.
Meanwhile, on December 18, 1917, Herrer's attorney, Enrique, wrote to the Manila Office
stating that Herrer wanted to withdraw his application to which the office wrote a letter dated
November 26, 1917 stating that the policy had already been issued. The letter was received by
the attorney on December 21, 1917. Herrer had died a day earlier on December 20, 1920.

The trial court ruled that the contract had been perfected, hence this appeal.

ISSUES:

1. Whether or not the policyholder had received notice of the acceptance of his policy;
2. Whether or not the contract of life annuity was perfected.

HELD:

1. No. The facts clearly show that Herrer was not informed of the acceptance of the policy before
his death.

2. No. The contract was not perfected. Art. 1262 provides that acceptance by letter does not bind
the person making the offer except from the time it came to his knowledge. The pertinent fact is
that according to the provisional receipt, the insurance company had to:

1) conduct a medical examination;

2) had to obtain the head office's approval; and

3) somehow communicate such approval.


It is true that the letter notifying acceptance was deposited in the post office, but the fact of
notification is a rebuttable presumption and the facts clearly show that Herrer never received the
notice of the acceptance before his death.

b. general concept of insurance


2. White Gold Marine Services, Inc. vs. Pioneer Insurance and Surety Corp., GR 154514, July
28, 2005

FACTS:

White Gold, petitioner, procured a protection and indemnity for its vessel from the
Steamship Mutual Underwriting Association through Pioneer Insurance and Security
Corporation. Subsequently, White Gold was issued a Certificate of Entry and Acceptance. When
petitioner failed to fully pay its account, Steamship Mutual refused to renew the coverage.

Steamship thereafter filed a case of collection of sum of money for the unpaid balance of
the petitioner while the latter filed before the Insurance Commissioner a case against Steamship
for violating Sections 186 and 187 of the Insurance Code, while Pioneer violated Sections 299,
300 and 301 in relation to Sections 302 and 303, thereof.

The Insurance Commissioner dismissed the complaint and said that there is no need for
the Steamship Mutual to procure license because it was not engage in insurance business and
was only a protection and indemnity club. Likewise, it ruled that Pioneer need not secure another
license as an insurance agent and/or a broker of Steamship Mutual because it was not engaged in
insurance business and Pioneer already had a license hence procurement of separate license as an
insurance agent would only be superfluous. CA affirmed the decision of Insurance
Commissioner.

ISSUE/S:

1) Whether or not Steamship Mutual, a P & I Club, is engaged in the insurance business
in the Philippines.

2) Whether or not Pioneer need another license as an insurance agent/broker for


Steamship Mutual.

RULING:

1) Yes, Steamship Mutual, a P & I Club, is engaged in the insurance business in the
Philippines.

The test to determine if a contract is an insurance contract or not, depends on the nature
of the promise, the act required to be performed, and the exact nature of the agreement in the
light of the occurrence, contingency, or circumstances under which the performance becomes
requisite. It is not by what it is called. In particular, a marine insurance undertakes to indemnify
the assured against marine losses, such as the losses incident to a marine adventure. Section 99 of
the Insurance Code enumerates the coverage of marine insurance.

Relatedly, a mutual insurance company is a cooperative enterprise where the members


are both the insurer and insured. In it, the members all contribute, by a system of premiums or
assessments, to the creation of a fund from which all losses and liabilities are paid, and where the
profits are divided among themselves, in proportion to their interest. Additionally, mutual
insurance associations, or clubs, provide three types of coverage, namely, protection and
indemnity, war risks, and defense costs.

A P & I Club is a form of insurance against third party liability, where the third party is
anyone other than the P & I Club and the members.[19] By definition then, Steamship Mutual as
a P & I Club is a mutual insurance association engaged in the marine insurance business.

The records reveal Steamship Mutual is doing business in the country albeit without the
requisite certificate of authority mandated by Section 187[20] of the Insurance Code. It maintains
a resident agent in the Philippines to solicit insurance and to collect payments in its behalf. We
note that Steamship Mutual even renewed its P & I Club cover until it was cancelled due to non-
payment of the calls. Thus, to continue doing business here, Steamship Mutual or through its
agent Pioneer, must secure a license from the Insurance Commission.

Since a contract of insurance involves public interest, regulation by the State is necessary.
Thus, no insurer or insurance company is allowed to engage in the insurance business without a
license or a certificate of authority from the Insurance Commission.

2) Yes, Pioneer need another license as an insurance agent/broker for Steamship Mutual.

The court ruled that although Pioneer is already licensed as an insurance company, it
needs a separate license to act as insurance agent for Steamship Mutual. Section 299 of the
Insurance Code clearly states:

“No person shall act as an insurance agent or as an insurance broker in the solicitation or
procurement of applications for insurance, or receive for services in obtaining insurance, any
commission or other compensation from any insurance company doing business in the
Philippines or any agent thereof, without first procuring a license so to act from the
Commissioner, which must be renewed annually on the first day of January, or within six months
thereafter.”.

3. Philippine Health Care Providers, Inc. vs. Commissioner of Internal Revenue, GR No.
167330, September 18, 2009

FACTS:
Petitioner Phil. Healthcare providers asked for a motion for reconsideration from the SC
ruling that they were engaged in health insurance, contrary to their claim of being a health
management organization (HMO).
Phil. Healthcare Providers is a domestic corporation that provides preventive, diagnostic,
and curative services to members who pay them a healthcare membership whether they use the
services or not, but also pay for any expenses incurred by its members.
The CIR issued them a tax deficiency on documentary stamp taxes, on the basis of NIRC
Sec. 185, along with surcharges, The tax deficiency amounted to P224,702,641.18.
Petitioners argue that the healthcare management agreement is not one of insurance, but
for the provision of healthcare services, that the primary goal of the agreement is the provision of
medical services, and not the indemnification of the members should they fall ill.

ISSUE/S:
Whether or not Phil. Healthcare Providers were engaged in the contract of insurance.

RULING:
No, the petitioners are not in the business of insurance. The test in determining the
presence of an insurance contract is not mere assumption of risk, as almost all business
obligations have an assumption of risk, but by the principal object and purpose test.
In this case, what is the principal object and purpose of the health management contract?
It is the provision of affordable healthcare, and not the mitigation of risk of the members. The
lower costs to the members who avail of the plan is merely incidental to the contract.
Mere assumption of risk by the HMO is not enough to override the principal object of the
contract.

4. Philamcare Health System vs. CA, 379 SCRA 356

Facts:

Ernani Trinos, deceased husband of private respondent Julita Trinos, was approved
for a health care coverage with petitioner from March 1988 to March1989. The same was
extended twice until June 1990. During the period of his coverage, Ernani was hospitalized
several times, however, petitioner denied the claim of private respondent because the
Health Care Agreement was allegedly void due to the alleged concealment of Ernani that
he was not hypertensive, diabetic, and asthmatic, contrary to his answer in the application
form.
Petitioner argues that the agreement merely granted living benefits, such as check-
ups and hospitalisation, hence it is not an insurance contract. Petitioner further argues that
it is not an insurance company, which is governed by the Insurance Commission, but a
Health Maintenance Organization under the authority of the Department of Health.

Issues:

1. Whether or not the Health Care Agreement between the deceased and the
petitioner falls under the ambit of an insurance contract.

2. Whether the alleged concealment of the deceased will invalidate the Agreement.

Ruling:

1. Yes. In the case at bar, the insurable interest of respondents husband in obtaining
the health care agreement was his own health. Section 10 of the Insurance Code is clear
that every person has an insurable interest in the life and health of himself. The health
care agreement was in the nature of non-life insurance, which is primarily a contract of
indemnity. Once the member incurs hospital, medical or any other expense arising from
sickness, injury or other stipulated contingent, the health care provider must pay for
the same to the extent agreed upon under the contract.

2. No. The answer assailed by petitioner was in response to the question relating to the
medical history of the applicant. This largely depends on opinion rather than fact,
especially coming from respondents husband who was not a medical doctor. Where
matters of opinion or judgment are called for, answers made in good faith and without
intent to deceive will not avoid a policy even though they are untrue. (A)lthough false, a
representation of the expectation, intention, belief, opinion, or judgment of the insured
will not avoid the policy if there is no actual fraud in inducing the acceptance of the
risk, or its acceptance at a lower rate of premium, and this is likewise the rule although
the statement is material to the risk, if the statement is obviously of the foregoing
character, since in such case the insurer is not justified in relying upon such statement,
but is obligated to make further inquiry. There is a clear distinction between such a
case and one in which the insured is fraudulently and intentionally states to be true, as a
matter of expectation or belief, that which he then knows, to be actually untrue, or the
impossibility of which is shown by the facts within his knowledge, since in such case the
intent to deceive the insurer is obvious and amounts to actual fraud. Under Section 27
of the Insurance Code, a concealment entitles the injured party to rescind a contract of
insurance. The right to rescind should be exercised previous to the commencement of
an action on the contract.

5. Lalican vs. Insular Life Assurance Co., Ltd. 597 SCRA 159

FACTS:
Violeta is the widow of Eulogio C. Lalican. Eulogio applied for an insurance policy with Insular
Life through its agent Malaluan. Policy No. 9011992 was issued to Eulogio, it contained a 20-
Year Endowment Variable Income Package Flexi Plan and Violeta was named as the primary
beneficiary.

Under the terms of the policy, Eulogio was to pay premiums on a quarterly basis every 24 April,
24 July, 24 October and 24 January of each year, until the end of the 20-year period of the
policy. There was a grace period of 31 days for the payment of each premium and if the premium
was not paid on the due date, then the policy would be in default, and if the premium remained
unpaid until the end of the grace period, the policy would become void.

Eulogio failed to pay the premium due on Jan 24 1998, even after the lapse of the grace period,
hence the period became void. Eulogio submitted to Insular Life an Application of Reinstatement
but it could not be fully processed because there was an overdue interest amounting to P322.

Eulogio went to Malaluan's house to submit a second Application for Reinstatement including
the payments. However on the same day Eulogio died of cardiac arrest.

Malaluan forwarded the application to Insular Life the next day but Insular Life no longer acted
upon the application as Eulogio had already passed away.

Violeta filed with the RTC a Complaint for Death Claim Benefit, the RTC rendered a Decision
in favor of Insular Life, hence this petition.

ISSUE:
Whether Eulogio was able to reinstate the lapsed insurance policy on his life before his death

RULING:
No. The policy had already lapsed. Eulogio's filing of an Application of Reinstatement
constitutes an admission that the policy had already lapsed.
Eulogio's death made it impossible to fully comply with the conditions of reinstatement in Policy
No. 9011992. The policy could only be considered reinstated after it had been processed and
approved by Insular Life during Eulogio's lifetime and good health. Only Insular Life had the
power to reinstate the policy, it does not matter that the application was with Malaluan before
Eulogio died.

6. Alpha Insurance and Surety Co. vs. Castor, GR 198174, September 2, 2013

Facts: On February 21, 2007, respondent entered into a contract of insurance, Motor Car Policy
No. MAND/CV-00186, with petitioner, involving her motor vehicle, a Toyota Revo DLX DSL.
The contract of insurance obligates the petitioner to pay the respondent the amount of Six
Hundred Thirty Thousand Pesos (P 630,000.00) in case of loss or damage to said vehicle during
the period covered, which is from February 26, 2007 to February 26, 2008. On April 16, 2007, at
about 9:00 a.m., respondent instructed her driver, Jose Joel Salazar Lanuza (Lanuza), to bring the
above-described vehicle to a nearby auto-shop for a tune-up. However, Lanuza no longer
returned the motor vehicle to respondent and despite diligent efforts to locate the same, said
efforts proved futile. Resultantly, respondent promptly reported the incident to the police and
concomitantly notified petitioner of the said loss and demanded payment of the insurance
proceeds in the total sum of P 630,000.00. In a letter dated July 5, 2007, petitioner denied the
insurance claim of respondent, stating among others, thus: Upon verification of the documents
submitted, particularly the Police Report and your Affidavit, which states that the culprit, who
stole the Insured unit, is employed with you. We would like to invite you on the provision of the
Policy under Exceptions to Section-III.

Issue: Whether or not respondent Castor is entitled to the insurance policy for the loss of her car
by her driver.

Ruling: Yes. It is a basic rule in the interpretation of contracts that the terms of a contract are to
be construed according to the sense and meaning of the terms which the parties thereto have
used. In the case of property insurance policies, the evident intention of the contracting parties,
i.e., the insurer and the assured, determine the import of the various terms and provisions
embodied in the policy. However, when the terms of the insurance policy are ambiguous,
equivocal or uncertain, such that the parties themselves disagree about the meaning of particular
provisions, the policy will be construed by the courts liberally in favor of the assured and strictly
against the insurer.

A contract of insurance is a contract of adhesion. So, when the terms of the insurance contract
contain limitations on liability, courts should construe them in such a way as to preclude the
insurer from non-compliance with his obligation.
Theft perpetrated by the driver of the insured is not an exception to the coverage from the
insurance policy, since Section III thereof did not qualify as to who would commit the theft.

Therefore, petitioner cannot exclude the loss of respondent’s vehicle under the insurance policy
under paragraph 4 of “Exceptions to Section III,” since the same refers only to “malicious
damage,” or more specifically, “injury” to the motor vehicle caused by a person under the
insured’s service. Paragraph 4 clearly does not contemplate “loss of property,” as what happened
in the instant case.

c. characteristics

1. risk distributing device ---

7. UCPB General Insurance vs. Masagana Telemart, GR 137172, April 4,2001

FACTS:

Masagana Telemart (Masagana for brevity) obtained 5 insurance policies, effective from 22 May
1991 to 22 May 1992, on its properties from UCPB Gen. Insurance Co. (UCPB for brevity).
Nonetheless, prior to said insurance policies, Masagana had procured insurance coverage from
UCPB for a number of years, all granted a 60 to 90-day credit term within which to pay the
premiums on the renewed policies.

On 13 June 1992, Masagana’s properties were razed by fire. On the same day, or within the
implied 60 to 90-day credit term, Masagana renewed the insurance policies and paid the
premiums, to which UCPB accepted. On 14 June 1992, Masagana made its formal demand for
indemnification for the burned insured properties, but the same was rejected by UCPB for the
reason that the subject insurance policies expired on May 22 but the fire took place on June 13,
or before tender of premium payment for the renewed insurance policies.

Masagana, on the one hand, argues that parties may either agree expressly or impliedly on the
extension of credit or time to pay the premium or consider a policy binding before actual
payment. Despite the mandate of Sec. 77,[1] extension of credit terms in premium payment has
been the prevalent practice in the insurance industry. Moreover, Sec. 78 authorizes the validity
of a policy notwithstanding non-payment of premiums. UCPB, on the others, asserts that
Masagana’s argument that Sec. 77 is not a prohibitive provision finds no authoritative support.
[1] Sec. 77. “An insurer is entitled to payment of the premium as soon as the thing
insured is exposed to the peril insured against. Notwithstanding any agreement to the contrary,
no policy or contract of insurance issued by an insurance company is valid and binding unless
and until the premium thereof has been paid, except in the case of a life or an industrial life
policy whenever the grace period provision applies.” (emphasis supplied by MJ Esteban).

ISSUE/S:

Whether Sec. 77 of the Insurance Code must be strictly applied to petitioner’s advantage despite
its practice of granting 60 to 90-day credit term for the payment of premiums.

RULING:

The Court answered in the negative.

While Sec. 77 is he general rule (i.e., “no policy or contract of insurance issued by an insurance
company is valid and binding unless and until the premium thereof has been paid”), the Court
said that it allows exceptions:

1. In case of life or industrial life policy whenever the grace period provision applies;

2. Sec. 78 (now Sec. 79[1]);

3. If the parties have agreed to the payment in installments of the premium and partial
payment has been made at the time of loss (Makati Tuscany Condominium Corp. v CA); and

4. That the insurer may grant credit extension for the payment of the premium (Id.).

This simply means, pertaining to the fourth exemption, that if the insurer has granted the insured
a credit term for the payment of the premium and loss occurs before the expiration of the same
credit term, recovery on the policy should be allowed even though the premium is paid after the
loss but within the credit term

Moreover, by virtue of autonomy provided under Art. 1306 of the Civil Code, nothing in Sec. 77
which prohibits the parties in an insurance contract in providing a credit term within which to
pay the premiums. It would be unjust and inequitable if recovery on the policy would not be
permitted against UCPB, which had consistently granted a 60 to 90-day credit term for the
payment of premiums despite its full awareness of Sec. 77. Estoppel bars it from taking refuge
under said Section, since Masagana relied in good faith on such practice.
[1] Sec. 79. “Any acknowledgment in a policy or contract of insurance of the receipt of premium
is conclusive evidence of its payment, so far as to make the policy binding, notwithstanding any
stipulation therein that it shall not be binding until premium is actually paid.”

SEPARATE OPINION OF JUSTICE VITUG

The insurer assumes the risk of loss which an insured might suffer in consideration of premium
payments under a risk-distributing device. Such assumption of risk is a component of a general
scheme to distribute actual losses among a group of persons, bearing similar risks, who make
contributions to a fund from which the losses incurred due to exposures to the peril insured
against are assured and compensated. A requirement imposed by way of State regulation upon
insurers is the maintenance of an adequate legal reserve in favor of those claiming under their
policies. The law generally mandates that insurance companies should retain an amount
sufficient to guarantee the security of its policyholders in the remote future, as well as the
present, and to cover aNy contingencies that may arise or may be fairly anticipated.

Calculations and estimations of liabilities under the risk insured against are predicated based on
the payment of premiums, the vital element that establishes the juridical relation between the
insured and the insurer. By legislative fiat, any agreement to the contrary notwithstanding, the
payment of premium is a condition precedent to, and essential for, the efficaciousness of the
insurance contract, subject to exceptions.

2. contract of adhesion ---


8. Rizal Surety & Insurance Co. vs. CA, GR 112360, July 18, 2000

FACTS:

Rizal Surety & Insurance Company (Rizal Insurance) issued Fire Insurance Policy No. 45727 in
favor of Transworld Knitting Mills, Inc. (Transworld).

Pertinent portions of subject policy on the buildings insured, and location thereof, read:

"‘On stocks of finished and/or unfinished products, raw materials and supplies of every kind
and description, the properties of the Insureds and/or held by them in trust, on commission or on
joint account with others and/or for which they (sic) responsible in case of loss whilst contained
and/or stored during the currency of this Policy in the premises

occupied by them forming part of the buildings situate (sic) within own Compound at
MAGDALO STREET, BARRIO UGONG, PASIG, METRO MANILA, PHILIPPINES, BLOCK
NO.601.’

x xx............ ...xxx....... ........xxx

Said building of four-span lofty one storey in height with mezzanine portions is constructed of
reinforced concrete and hollow blocks and/or concrete under galvanized iron roof and occupied
as hosiery mills, garment and

lingerie factory, transistor-stereo assembly plant, offices, warehouse and caretaker's quarters.
Bounds in front partly by one-storey concrete building under galvanized iron roof occupied as
canteen and guardhouse, partly by building of two and partly one storey constructed of concrete
below, timber above under galvanized iron roof occupied as garage and quarters and partly by
open space and/or tracking/ packing, beyond which is the aforementioned Magdalo Street; on its
right and left by driveway, thence open spaces, and at the rear by open spaces.'" The same pieces
of property insured with the petitioner were also insured with New India Assurance Company,
Ltd., (New India).

Fire broke out in the compound of Transworld, razing the middle portion of its four-span
building and partly gutting the left and right sections thereof. A two-storey building (behind said
fourspan building) where fun and amusement machines and spare parts were stored, was also
destroyed by the fire. Transworld filed its insurance claims with Rizal Surety & Insurance
Company and New India Assurance Company but to no avail.

Private respondent brought against the said insurance companies an action for collection of sum
of money and damages.

Petitioner Rizal Insurance countered that its fire insurance policy sued upon covered only the
contents of the four-span building, which was partly burned, and not the damage caused by the
fire on the two-storey annex building.

The trial court dismissed the case as against The New India Assurance Co., Ltd. but ordered
defendant Rizal Surety And Insurance Company to pay Transwrold (sic) Knitting Mills, Inc.
Both the petitioner, Rizal Insurance Company, and private respondent, Transworld Knitting
Mills, Inc., went to the Court of Appeals, which required New India Assurance Company to pay
plaintiff appellant the amount of P1,818,604.19 while the Rizal Surety has to pay the plaintiff-
appellant P470,328.67. New India appealed to the Court theorizing inter alia that the private
respondent could not be compensated for the loss of the fun and amusement machines and spare
parts stored at the two-storey building because it (Transworld) had no insurable interest in said
goods or items.

The Court denied the appeal with finality. Petitioner Rizal Insurance and private respondent
Transworld, interposed a Motion for Reconsideration before the Court of Appeals, which
reconsidered its decision of July 15, 1993, as regards the imposition of interest. Undaunted,
petitioner Rizal Surety & Insurance Company found its way to the Court.

ISSUE:

WON the fire insurance policy litigated upon protected only the contents of the main building
(four-span), and did not include those stored in the two-storey annex building

RULING:

NO. Resolution of the issue posited hinges on the proper interpretation of the stipulation
in subject fire insurance policy regarding its coverage, which reads:

"xxx contained and/or stored during the currency of this Policy in the premises occupied by them
forming part of the buildings situate (sic) within own Compound xxx"

It can be gleaned unerringly that the fire insurance policy in question did not limit its coverage to
what were stored in the four-span building. As opined by the trial court of origin, two
requirements must concur in order that the said fun and amusement machines and spare parts
would be deemed protected by the fire insurance policy under scrutiny, to wit:
"First, said properties must be contained and/or stored in the areas occupied by Transworld and
second, said areas must form part of the building described in the policy xxx"

Said building of four-span lofty one storey in height with mezzanine portions is constructed of
reinforced concrete and hollow blocks and/or concrete under galvanized iron roof and occupied
as hosiery mills, garment and lingerie factory, transistor-stereo assembly plant, offices, ware
house and caretaker's quarter.

The Court is mindful of the well-entrenched doctrine that factual findings by the Court of
Appeals are conclusive on the parties and not reviewable by this Court, and the same carry even
more weight when the Court of Appeals has affirmed the findings of fact arrived at by the lower
court.

In the case under consideration, both the trial court and the Court of Appeals found that the so
called "annex " was not an annex building but an integral and inseparable part of the four-span
building described in the policy and consequently, the machines and spare parts stored therein
were covered by the fire insurance in dispute. Verily, the two-storey building involved, a
permanent structure which adjoins and intercommunicates with the "first right span of the lofty
storey building", formed part thereof, and meets the requisites for compensability under the fire
insurance policy sued upon.

So also, considering that the two-storey building aforementioned was already existing when
subject fire insurance policy contract was entered into, petitioner should have specifically
excluded the said two-storey building from the coverage of the fire insurance if minded to
exclude the same but if did not, and instead, went on to provide that such fire insurance policy
covers the products, raw materials and supplies stored within the premises of respondent
Transworld which was an integral part of the four-span building occupied by Transworld,
knowing fully well the existence of such building adjoining and intercommunicating with the
right section of the four-span building.

Indeed, the stipulation as to the coverage of the fire insurance policy under controversy has
created a doubt regarding the portions of the building insured thereby. Article 1377 of the New
Civil Code provides:
"Art.1377. The interpretation of obscure words or stipulations in a contract shall not favor the
party who caused the obscurity"

Conformably, it stands to reason that the doubt should be resolved against the petitioner, Rizal

Surety Insurance Company, whose lawyer or managers drafted the fire insurance policy contract
under scrutiny. Citing the aforecited provision of law in point, the Court in Landicho vs.
Government Service Insurance System, ruled:

"This is particularly true as regards insurance policies, in respect of which it is settled that the
'terms in an insurance policy, which are ambiguous, equivocal, or uncertain x x x are to be
construed strictly and most strongly against the insurer, and liberally in favor of the insured so as
to effect the dominant purpose of indemnity or payment to the insured, especially where
forfeiture is involved' and the reason for this is that the 'insured usually has no voice in the
selection or arrangement of the words employed and that the language of the contract is selected
with great care and deliberation by experts and legal advisers employed by, and acting
exclusively in the interest of, the insurance company.”

9. Fortune Insurance & Surety Co. vs. CA, GR 115278, May 23, 1995

Facts:

Private respondent Producers Bank of the Philippines was insured by petitioner Fortune
Insurance and Safety Co., Inc. under its Money, Security, and Payroll Robbery policy. On June
29, 1987, an armored car of the private respondent bank, driven by Benjamin Magalong and
escorted by Saturnino Atiga, was robbed along Taft Avenue, Pasay City. After an investigation
by the Pasay police authorities, Magalong and Atiga were charged with violation of P.D. 532
(Anti-Highway Robbery Law) before the Fiscal of Pasay City who then filed an information
charging the aforesaid persons with the said crime before the RTC of Pasay City.

Demands were made by Producers Bank upon Fortune Insurance and Surety Co., Inc. to
pay the amount of the loss of P725,000, to which the latter refused to pay alleging that the loss is
excluded from the coverage of the insurance policy which states in its General Exceptions clause
that the company shall not be liable in respect of any loss caused by any dishonest, fraudulent or
criminal act of the insured or any officer, employee, partner, director, trustee or authorized
representative of the insured whether acting alone or in conjunction with others. Producers Bank
opposed the same stating that Atiga and Magalong are not its officer, employee, trustee, nor
authorized representative at the time of the robbery.
Issue:

Whether petitioner Fortune Insurance and Surety Co., Inc. was liable under the insurance
policy

Ruling:

No, Fortune Insurance and Surety Co., Inc. was not liable.

The Supreme Court ruled that a contract of insurance is a contract of adhesion, thus any
ambiguity therein should be resolved against the insurer, or it should be construed liberally in
favor of the insured and strictly against the insurer. Limitations of liability should be regarded
with extreme jealousy and must be construed in such a way as to preclude the insurer from
noncompliance with its obligation. It goes without saying then that if the terms of the contract
are clear and unambiguous, there is no room for construction and such terms cannot be enlarged
or diminished by judicial construction. An insurance contract is also a contract of indemnity
upon the terms and conditions specified therein. It is settled that the terms of the policy constitute
the measure of the insurer’s liability. In the absence of statutory prohibition to the contrary,
insurance companies have the same rights as individuals to limit their liability and to impose
whatever conditions they deem best upon their obligations not inconsistent with public policy.
With the foregoing principles in mind, it may now be asked whether Magalong and Atiga qualify
as employees or authorized representatives of Producers under the general exceptions clause of
the policy. Magalong and Atiga were, in respect of the transfer of Producers money from its
Pasay City branch to its head office in Makati, its “authorized representatives” who served as
such with its teller Maribeth Alampay. Howsoever viewed, Producers entrusted the three with
the specific duty to safely transfer the money to its head office, with Alampay to be responsible
for its custody in transit; Magalong to drive the armored vehicle which would carry the money;
and Atiga to provide the needed security for the money, the vehicle, and his two other
companions. In short, for these particular tasks, the three acted as agents of Producers. A
“representative” is defined as one who represents or stands in the place of another; one who
represents others or another in a special capacity, as an agent, and is interchangeable with
“agent.”

Therefore, Fortune Insurance and Surety Co., Inc. was not liable.

10. Gulf Resorts & Philippine Charter Insurance Corp., GR 156167, 5/16/05

FACTS:

Gulf Resorts Inc. owns a Plaza Resort in La Union. The properties were originally
insured with American Home Assurance Company. The first four insurance policies provided
that the risk of loss from earthquake shock was extended only to two swimming pools, thus,
“earthquake shock”. Item 5 in those policies referred to the two swimming pools only. When the
insurance was renewed for March 14, 1989-March 14, 1990, the earthquake endorsement clause
was deleted.
Gulf agreed to insure the same for March 14, 1990 to March 14, 1991 to Philippine
Charter Insurance Corporation. The questioned policy in this case provides that:
“In consideration of the payment by the insured to the company of the sum included additional
premium the Company agrees, notwithstanding what is stated in the printed conditions of this
policy due to the contrary, that this insurance covers loss or damage to shock to any of the
property insured by this Policy occasioned by or through or in consequence of earthquake.”
The word “included” was deleted. On July 16, 1990, an earthquake occurred. The
properties of Gulf Resorts in Plaza Resort including the two swimming pools were damaged.
Gulf advised Phil Charter that it will claim for the damages of its properties. A.R. De Leon, an
independent claims adjuster, made the report. The report stated that: “except for the swimming
pools, all affected items have no coverage for earthquake shocks.” The two parties failed to
arrive at a settlement. Gulf Resorts filed a complaint. The lower court ruled that the insurance
company was correct for the policy only covered the two swimming pools.

Gulf Resorts argued that the policy’s earthquake shock endorsement clearly covers all of
the properties insured and not only the swimming pools. It relied on the statement in the policy
saying: “any property insured by this policy.” Gulf Resorts also argued that the insurance
contract is a contract of adhesion. Having ambiguities, the insurance contract should be
construed in its favor.

The insurance company argued that none of the previous policies issued by American
Home Assurance extended the earthquake shock coverage to other properties except that of the
two swimming pools. It also argued that the premium paid showed that the amount 393.00
covered the earthquake shocks on the two swimming pools. This was supported by evidence.
Lastly, it argued that the deletion of the phrase pertaining to the limitation of the earthquake
shock endorsement to the two swimming pools in the policy schedule did not expand the
earthquake shock coverage to all of Gulf’s properties.

ISSUE: Whether or not the Phil Charter should indemnify all the properties of Gulf Resorts
damaged by the earthquake.

RULING: Only the two swimming pools were specified and covered. The premiums paid by
Gulf Resorts clearly proved that only the two swimming pools were covered by the policy.
According to the Supreme Court, a careful examination of the premium recapitulation will show
that it is the clear intent of the parties to extend the coverage only to the two swimming pools.
A contract of insurance exists when the following elements are present: a.) the insured
has insurable interest; b.) the insured is subject to risk of loss by the happening of the event; c.)
the insurer assumes risk; d.) Such assumption of risk is part of a general scheme to distribute
actual losses among large group of persons bearing a similar risk; e.) In consideration of the
insurer’s promise, the insured pays a premium. The insurance premium is the consideration paid
an insurer for undertaking to indemnify the insured against a specified peril. Based on the paid
premiums, it was clearly shown that no premiums were paid with regard to earthquake shock
coverage except on the two swimming pools.

On Gulf’s argument that it is a contract of adhesion and should be construed in its favor.
The Supreme Court opined that Gulf cannot rely on the general rule on contracts of adhesion.
The Court only applies such rule when the contract of adhesion is one-sided. For the Supreme
Court, it will only rule out blind adherence to terms where facts and circumstances will show that
they are basically one-sided. In this case, Gulf Insurance cannot claim that it did not know the
provisions of the policy. From the inception of the policy, Gulf had required Philippine Charter
to copy verbatim the provisions and terms of its latest insurance policy from American Home
Assurance Company.

11. Eternal Gardens Memorial Park vs. Philam Life, GR 166245, 4/9/08

FACTS:
Philamlife and Eternal Gardens entered into a Group Life Insurance underwhich the
clients of Eternal who purchased burial lots from it on installment basis would be insured by
Philamlife. Eternal was required under the policy to submit a list of new lot purchasers, together
with a copy of the application of each purchasers and the amounts of their respective unpaid
balances. Eternal sent a letter dated December 29, 1982 containing the list of insurable balances.
One of those included in the list was John Chuang. Philamlife did not reply to said letter. On
August 2, 1984, John Chuang died. Eternal demanded payment from Philamlife of the insurance
claim for John Chuang’s death, but Philamlife denied the claim on the ground that no application
for group insurance was submitted prior to John Chuang’s death.

It was provided in the relevant provision in the contract, particularly the “Effective Date
of Benefit” – “The insurance of any eligible lot purchaser shall be effective on the date he
contracts a loan with the Assured. However, there shall be no insurance if the application of the
lot purchaser is not approved by the Company.”

ISSUE/S:
Whether or not there was a valid contract of insurance covering Chuang’s life
considering the conflicting provisions of the policy
RULING:
Yes. There was a valid contract of insurance covering Chuang’s life.

An insurance contract is a contract of adhesion which must be construed liberally in favor


of the insured and strictly against the insurer in order to safeguard the latter’s interest. In
Malayan Insurance Corporation v. Court of Appeals, the court held that: A contract of insurance,
being a contract of adhesion, par excellence, any ambiguity therein should be resolved against
the insurer. Further, in Philamcare Health Systems, Inc. v. Court of Appeals: By reason of the
exclusive control of the insurance company over the terms and phraseology of the insurance
contract, ambiguity must be strictly interpreted against the insurer and liberally in favor of the
insured, especially to avoid forfeiture.

Hence, the vague contractual provision, in Creditor Group Life Policy must be construed
in favor of the insured and in favor of the effectivity of the insurance contract. On the other hand,
the seemingly conflicting provisions must be harmonized to mean that upon a party’s purchase of
a memorial lot on installment from Eternal, an insurance contract covering the lot purchaser is
created and the same is effective, valid, and binding until terminated by Philamlife by
disapproving the insurance application.

12. Manila Bankers’ Life vs. Aban, GR 175666, July 29, 2013

F A C T S:

On 3 July 1993, Delia Sotero (Sotero for brevity) took out a life insurance policy from Manila
Bankers Life Insurance Corp. (petitioner for brevity) naming Cresencia Aban (Aban for brevity).
On 10 April 1996, Sotero died, prompting Aban to file a claim for the insurance proceeds.
However, after conducting investigation, respondent denied such claim for reasons of fraud and
concealment, arguing that Aban was the one who filed Sotero’s claim on 3 July 1993 and
designated herself as the beneficiary.

Petitioner then filed a case for recission and/or annulment of the policy. Aban countered with a
motion to dismissed, which the trial court granted. The RTC ruled that it was Sotero, not Aba,
who procured the insurance. Also, under Sec. 48 of the Insurance Code, petitioner had only 2
years from the effectivity of the insurance policy to question the same; petitioner is now barred
from contesting the policy. The CA affirmed RTC’s decision.

I S S U E:
Whether petitioner Manila Bankers Life Insurance is barred from denying the insurance claims
based on fraud or concealment.

H E L D:

Yes, the petitioner is barred from denying the insurance by reason of the “incontestability
clause” under Sec. 48[1] of the Insurance Code. The reason for such clause is that insurers who
recklessly and indiscriminately solicit and obtain business must be penalized, for such reckless
and lack of indiscrimination ultimately work to the detriment of bona fide takers of insurance
and public in general. It compels the insurers to solicit business from or provide insurance
coverage only to legitimate and bona fide clients, by requiring them to thoroughly investigate
those they insure within 2 years from effectivity of the policy and while the insured is still alive.
The law assumes that they will do just that and not sit on their laurels.

Sec. 48. “Whenever a right to rescind a contract of insurance is given to the insurer by
any provision of this chapter, such right must be exercised previous to the commencement of an
action on the contract. After a policy of life insurance made payable on the death of the insured
shall have been in force during the lifetime of the insured for a period of 2 years from the date of
its issue or of its last reinstatement, the insurer cannot prove that the policy is void ab initio or is
rescindable by reason of the fraudulent concealment or misrepresentation of the insured or his
agent.”

Sec. 48 regulates both the actions of the insurers and prospective takers of life insurance. It
gives insurers enough time to inquire whether the policy was obtained by fraud, concealment, or
misrepresentation; o the other hand, it forewarns scheming individuals that their attempts at
insurance fraud would be timely uncovered – thus deterring them from venturing into such
nefarious enterprise. At the same time, legitimate policy holders are protected from unwarranted
denial of their claims or delay in the collection of insurance proceeds occasioned by allegations
of fraud, concealment, or misrepresentation by insurers, claims which may no longer be set up
after the 2-year period expires.

Life insurance policies that pass the statutory 2-year period are essentially treated as legitimate
and beyond question, and the individual who wield them are made secure by the thought that
they will be paid promptly upon claim.

In the instant case, instead of investigating at the first instance, petitioner appears to have turned
a blind eye and opted instead to continue collecting the premiums on the policy. For nearly 3
years, petitioner collected the premiums and devoted the same to its own profit. Insurers cannot
be allowed to collect premiums on insurance policies, use these amounts collected and invest the
same through the years, generating profits and returns therefrom for their own benefit, and
thereafter conveniently deny insurance claims by questioning the integrity of such policy.

The business of insurance is a highly regulated commercial activity, and is imbued with public
interest. An insurance contract is a contract of adhesion which must be construed liberally in
favor of the insured and strictly against the insurer in order to safeguard the former’s interest.

3. aleatory
4. contract of indemnity ---
13. Verendia vs. CA, GR 75605, January 22, 1993

Facts:

Fidelity and Surety Insurance Company (Fidelity) issued Fire Insurance Policy No. F-18876
effective between June 23, 1980 and June 23, 1981 covering Rafael (Rex) Verendia's residential
building in the amount of P385,000.00. Verendia also insured the same building with two other
companies, namely, The Country Bankers Insurance for P56,000.00 and The Development
Insurance for P400,000.00. While the three fire insurance policies were in force, the insured
property was completely destroyed by fire. Fidelity appraised the damage amounting to 385,00.
Despite the demands of Verendia for insurance claim, Fidelity refused payment under its policy
stating that the former maliciously represented that the building at the time of the fire was leased
under a contract executed on June 25, 1980 to a certain Roberto Garcia, when actually it was a
Marcelo Garcia who was the lessee. Thus, Verendia filed a complaint with the CFI of Quezon
City which then ruled in favor of Fidelity stating that Verendia violated paragraph 3 of the policy
by in that the insured failed to inform Fidelity of his other insurance coverages with Country
Bankers Insurance and Development Insurance and that he misrepresented the lessee of said
building.

Issue:

Whether or not Verendia can claim on the insurance despite the misrepresentation as to
the lessee and the over-insurance.

Ruling:

No, Verendia cannot claim on the insurance despite the misrepresentation as to


the lessee and the over-insurance.
The Supreme Court ruled that the contract of lease upon which Verendia relies to support
his claim for insurance benefits, was entered into between him and one Robert Garcia, a couple
of days after the effectivity of the insurance policy. When the rented residential building was
razed to the ground, it appears that Robert Garcia was still within the premises. However,
according to the investigation by the police, the building appeared to have "no occupants" and
that Mr. Roberto Garcia was renting on the other side of said compound. Ironically, during the
trial, Verendia admitted that it was not Robert Garcia who signed the lease contract but it was
Marcelo Garcia cousin of Robert, who had also been paying the rentals all the while. Verendia,
however, failed to explain why Marcelo had to sign his cousin's name when he in fact he was
paying for the rent. The Court agrees with Fidelity's conclusions that Verendia concocted the
lease contract to deflect responsibility for the fire towards an alleged lessee, inflated the value of
the property by the alleged monthly rental.

Basically a contract of indemnity, an insurance contract is the law between the parties. Its
terms and conditions constitute the measure of the insurer's liability and compliance therewith is
a condition precedent to the insured's right to recovery from the. As it is also a contract of
adhesion, an insurance contract should be liberally construed in favor of the insured and strictly
against the insurer company which usually prepares it
Considering, however, the foregoing discussion pointing to the fact that Verendia used a false
lease contract to support his claim under Fire Insurance Policy, the terms of the policy should be
strictly construed against the insured. Verendia failed to live by the terms of the policy, having
presented a false declaration to support his claim for benefits in the form of a fraudulent lease
contract, he forfeited all benefits therein by virtue of Section 13 of the policy in the absence of
proof that Fidelity waived such provision.

14. Fortune Insurance and Surety vs. CA, GR 115278, May 23, 1995

Facts:

Private respondent Producers Bank of the Philippines was insured by petitioner Fortune
Insurance and Safety Co., Inc. under its Money, Security, and Payroll Robbery policy. On June
29, 1987, an armored car of the private respondent bank, driven by Benjamin Magalong and
escorted by Saturnino Atiga, was robbed along Taft Avenue, Pasay City. After an investigation
by the Pasay police authorities, Magalong and Atiga were charged with violation of P.D. 532
(Anti-Highway Robbery Law) before the Fiscal of Pasay City who then filed an information
charging the aforesaid persons with the said crime before the RTC of Pasay City.

Demands were made by Producers Bank upon Fortune Insurance and Surety Co., Inc. to
pay the amount of the loss of P725,000, to which the latter refused to pay alleging that the loss is
excluded from the coverage of the insurance policy which states in its General Exceptions clause
that the company shall not be liable in respect of any loss caused by any dishonest, fraudulent or
criminal act of the insured or any officer, employee, partner, director, trustee or authorized
representative of the insured whether acting alone or in conjunction with others. Producers Bank
opposed the same stating that Atiga and Magalong are not its officer, employee, trustee, nor
authorized representative at the time of the robbery.

Issue:

Whether petitioner Fortune Insurance and Surety Co., Inc. was liable under the insurance
policy

Ruling:

No, Fortune Insurance and Surety Co., Inc. was not liable.

The Supreme Court ruled that a contract of insurance is a contract of adhesion, thus any
ambiguity therein should be resolved against the insurer, or it should be construed liberally in
favor of the insured and strictly against the insurer. Limitations of liability should be regarded
with extreme jealousy and must be construed in such a way as to preclude the insurer from
noncompliance with its obligation. It goes without saying then that if the terms of the contract
are clear and unambiguous, there is no room for construction and such terms cannot be enlarged
or diminished by judicial construction. An insurance contract is also a contract of indemnity
upon the terms and conditions specified therein. It is settled that the terms of the policy constitute
the measure of the insurer’s liability. In the absence of statutory prohibition to the contrary,
insurance companies have the same rights as individuals to limit their liability and to impose
whatever conditions they deem best upon their obligations not inconsistent with public policy.
With the foregoing principles in mind, it may now be asked whether Magalong and Atiga qualify
as employees or authorized representatives of Producers under the general exceptions clause of
the policy. Magalong and Atiga were, in respect of the transfer of Producers money from its
Pasay City branch to its head office in Makati, its “authorized representatives” who served as
such with its teller Maribeth Alampay. Howsoever viewed, Producers entrusted the three with
the specific duty to safely transfer the money to its head office, with Alampay to be responsible
for its custody in transit; Magalong to drive the armored vehicle which would carry the money;
and Atiga to provide the needed security for the money, the vehicle, and his two other
companions. In short, for these particular tasks, the three acted as agents of Producers. A
“representative” is defined as one who represents or stands in the place of another; one who
represents others or another in a special capacity, as an agent, and is interchangeable with
“agent.”

Therefore, Fortune Insurance and Surety Co., Inc. was not liable.
5. uberrimae fides contract
15. Heirs of Loreto C. Maramag vs. Eva Verna De Guzman Maramag, et al., GR 181132, June
5, 2009
16. Tibay vs. CA, 257 SCRA 126

FACTS:
In January 22 1987, the Petitioner Violeta Tibay (and Nicolas Roralso) obtained a fire
insurance policy for their 2-storey from the Private Respondent Fortune Life Insurance Co. The
said policy covers the period from January 23, 1987 until January 23, 1988 or one year for P600,
000 and at the agreed premium of P2, 983.50. On January 23 or the next day, petitioner made a
partial payment of the premium with P600. Unfortunately, on March 8 1987, the said building
was burned to the ground. It was only two days after the fire that Petitioner Violeta advanced the
full payment of the policy premium which was accepted by the insurer. On this same day,
petitioner likewise filed the claim that was then referred to the insurer's adjuster. Investigation of
the cause of fire commenced and the petitioner submitted the required proof of loss.

Despite that, the private respondent Fortune refused to pay the insurance claim saying it
as not liable due to the non-payment by petitioner of the full amount of the premium as stated in
the policy. The trial court rule in favor of the petitioner. Upon appeal, the Court of Appeals
reversed the lower court's decision and held that Fortune is not liable but ordered it to return the
premium paid with interest to the petitioner. Hence, this petition for review.
ISSUE/S:
Whether or not the partial payment of the premium rendered the insurance policy
ineffective.

RULING:
Yes. Insurance is a contract whereby one undertakes for a consideration to
indemnify another against loss, damage or liability arising from an unknown or contingent
event. The consideration is the premium, which must be paid at the time, way and manner
as stated in the policy, and if not so paid as in this case, the policy is therefore forfeited by
its own terms. In this case, the policy taken out by the petitioner provides for payment of
premium in full. Since the petitioner only made partial payment with the remaining
balance paid only after the fire or peril insured against has occurred, the insurance
contract therefore did not take effect barring the insured from claiming or collecting from
the loss of her building.

Under Section 77 of the Insurance Code (Philippine), it provides therein that "An
insurer is entitled to payment of the premium as soon as the thing insured is exposed to the
peril insured against. Notwithstanding any agreement to the contrary, no policy or contract
of insurance issued by an insurance company is valid and binding unless and until the
premium thereof has been paid, except in the case of a life or an industrial life policy
whenever the grace period provision applies." Herein case, the controversy is on the
payment of the premium. It cannot be disputed that premium is the elixir vitae of the
insurance business because the insurer is required by law to maintain a reserve fund to
meet its contingent obligations to the public. Due to this, it is imperative that the premium
is paid fully and promptly. To allow the possibility of paying the premium even after the
peril has ensued will surely undermine the foundation of the insurance business.

d. elements of insurance
17. Philamcare Health System vs. CA, 379 SCRA 432

Facts:

Ernani Trinos, deceased husband of private respondent Julita Trinos, was approved
for a health care coverage with petitioner from March 1988 to March1989. The same was
extended twice until June 1990. During the period of his coverage, Ernani was hospitalized
several times, however, petitioner denied the claim of private respondent because the
Health Care Agreement was allegedly void due to the alleged concealment of Ernani that
he was not hypertensive, diabetic, and asthmatic, contrary to his answer in the application
form.

Petitioner argues that the agreement merely granted living benefits, such as check-
ups and hospitalisation, hence it is not an insurance contract. Petitioner further argues that
it is not an insurance company, which is governed by the Insurance Commission, but a
Health Maintenance Organization under the authority of the Department of Health.

Issues:

1. Whether or not the Health Care Agreement between the deceased and the
petitioner falls under the ambit of an insurance contract.

2. Whether the alleged concealment of the deceased will invalidate the Agreement.

Ruling:

1. Yes. In the case at bar, the insurable interest of respondents husband in obtaining
the health care agreement was his own health. Section 10 of the Insurance Code is clear
that every person has an insurable interest in the life and health of himself. The health
care agreement was in the nature of non-life insurance, which is primarily a contract of
indemnity. Once the member incurs hospital, medical or any other expense arising from
sickness, injury or other stipulated contingent, the health care provider must pay for
the same to the extent agreed upon under the contract.

2. No. The answer assailed by petitioner was in response to the question relating to the
medical history of the applicant. This largely depends on opinion rather than fact,
especially coming from respondents husband who was not a medical doctor. Where
matters of opinion or judgment are called for, answers made in good faith and without
intent to deceive will not avoid a policy even though they are untrue. (A)lthough false, a
representation of the expectation, intention, belief, opinion, or judgment of the insured
will not avoid the policy if there is no actual fraud in inducing the acceptance of the
risk, or its acceptance at a lower rate of premium, and this is likewise the rule although
the statement is material to the risk, if the statement is obviously of the foregoing
character, since in such case the insurer is not justified in relying upon such statement,
but is obligated to make further inquiry. There is a clear distinction between such a
case and one in which the insured is fraudulently and intentionally states to be true, as a
matter of expectation or belief, that which he then knows, to be actually untrue, or the
impossibility of which is shown by the facts within his knowledge, since in such case the
intent to deceive the insurer is obvious and amounts to actual fraud. Under Section 27
of the Insurance Code, a concealment entitles the injured party to rescind a contract of
insurance. The right to rescind should be exercised previous to the commencement of
an action on the contract.

18. Fortune Medicare Inc. vs. Amorin, GR 195872, March 12, 2014

FACTS: While Amorin was on vacation in Hawaii, he underwent an emergency surgery,


specifically an appendectomy, causing him to incur professional and hospitalization expenses of
US$7,242.35 and US$1,777.79, respectively. Being a cardholder/member of Fortune Medicare,
Inc. (Fortune Care), a corporation engaged in providing health maintenance services to its
members, attempted to recover the full amount upon his return to Manila. However, the company
merely approved a reimbursement of P12,151.36, an amount that was based on the average cost
of appendectomy, net of Medicare deduction, if the procedure were performed in an accredited
hospital in Metro Manila. Amorin received under protest the approved amount, but asked for its
adjustment to cover the total amount of professional fees which he had paid, and eighty percent
(80%) of the approved standard charges based on “American standard”, considering that the
emergency procedure occurred in the U.S.A., citing provisions of the contract.

He then filed a complaint for breach of contract with damages but this was dismissed by the
RTC. It said that the parties intended to use the Philippine standard as basis. However, the CA
reversed this. The appellate court pointed out that, first, health care agreements such as the
subject Health Care Contract, being like insurance contracts, must be liberally construed in favor
of the subscriber. In case its provisions are doubtful or reasonably susceptible of two
interpretations, the construction conferring coverage is to be adopted and exclusionary clauses of
doubtful import should be strictly construed against the provider. Second, the CA explained that
there was nothing under the Health Care Contract which provided that the Philippine standard
should be used even in the event of an emergency confinement in a foreign territory.

ISSUE/S:
1. Whether a member of a health care provider can recover to the extent agreed in the contract.

2. Whether ambiguities should be taken in favor of the member.

RULING:

1. Yes. In the case at bar, the Supreme Court said that for purposes of determining the liability
of a health care provider to its members, jurisprudence holds that a health care agreement
is in the nature of non-life insurance, which is primarily a contract of indemnity. Once the
member incurs hospital, medical or any other expense arising from sickness, injury or other
stipulated contingent, the health care provider must pay for the same to the extent agreed upon
under the contract.

2. Yes. With regard to the ambiguities in the contract, settled is the rule that they should be
interpreted against the party that caused the ambiguity. “Any ambiguity in a contract whose
terms are susceptible of different interpretations must be read against the party who drafted it.”
Furthermore, it affirmed the CA’s finding that Fortune Care’s liability to Amorin under the
subject Health Care Contract should be based on the expenses for hospital and professional fees
which he actually incurred, and should not be limited by the amount that he would have incurred
had his emergency treatment been performed in an accredited hospital in the Philippines.

19. Jaime T. Gaisano vs. Development Insurance and Surety Corporation, GR 190702, February
27, 2017

FACTS:

Intercapitol Marketing Corporation (IMC) is the maker of Wrangler Blue Jeans. while
Levi Strauss (Phils.) Inc. (LSPI) is the local distributor of products bearing trademarks owned by
Levi Strauss & Co.
IMC and LSPI separately obtained from Insurance Company of North America fire
insurance policies for their book debt endorsements related to their ready-made clothing
materials which have been sold or delivered to various customers and dealers of the Insured
anywhere in the Philippines which are unpaid 45 days after the time of the loss.

On February 25, 1991, Gaisano Superstore Complex in Cagayan de Oro City, owned by
Gaisano Cagayan, Inc., containing the ready-made clothing materials sold and delivered by IMC
and LSPI was consumed by fire.

On February 4, 1992, the Insurance Company of North America filed a complaint for
damages against Gaisano Cagayan, Inc. alleges that IMC and LSPI filed their claims under their
respective fire insurance policies which it paid thus it was subrogated to their rights.

In its counter claim, Gaisano Cagayan, Inc contends that it cannot be held liable because
it was destroyed due to fortuities event or force majeure.

The RTC dismissed respondent’s complaint. It held that fire was not attributable to the
negligence of the petitioner and that IMC and LSPI retained ownership of the delivered goods
until fully paid, it must bear the loss (res perit domino).

Dissatisfied, petitioner appealed to the CA who rendered its decision setting aside the
decision of the RTC.

ISSUE/S:

Whether or not there is an insurable interest on book debts.

RULING:

YES. Section 13 of our Insurance Code defines insurable interest as "every interest in
property, whether real or personal, or any relation thereto, or liability in respect thereof, of such
nature that a contemplated peril might directly damnify the insured." Parenthetically, under
Section 14 of the same Code, an insurable interest in property may consist in:

(a) an existing interest;

(b) an inchoate interest founded on existing interest; or

(c) an expectancy, coupled with an existing interest in that out of which the expectancy arises.

Therefore, an insurable interest in property does not necessarily imply a property interest
in, or a lien upon, or possession of, the subject matter of the insurance, and neither the title nor a
beneficial interest is requisite to the existence of such an interest, it is sufficient that the insured
is so situated with reference to the property that he would be liable to loss should it be injured or
destroyed by the peril against which it is insured. Anyone has an insurable interest in property
who derives a benefit from its existence or would suffer loss from its destruction. Indeed, a
vendor or seller retains an insurable interest in the property sold so long as he has any interest
therein, in other words, so long as he would suffer by its destruction, as where he has a vendor's
lien. In this case, the insurable interest of IMC and LSPI pertain to the unpaid accounts appearing
in their Books of Account 45 days after the time of the loss covered by the policies, and the
petitioner is liable for such unpaid accounts.

II. Contract of Insurance


a. requisites of a contract of insurance
b. perfection
c. parties to a contract of insurance
1. minor
2. married woman
3. mortgagee/mortgagor
4. assignee
d. subject matter of insurance
e. insurance not a wagering contract

 Enriquez vs. Sun Life Insurance of Canada (G.R. No. 15895, Nov. 29, 1920);

FACTS:

Joaquin Herrer made an application with Sun Life for a life annuity. He paid the amount of
P6,000.00 to the Manila manager who gave him a "provisional" receipt "subject to medical
examination and approval of the Company's Central Office." The application was forwarded to
the head office in Canada and the policy was issued on December 4, 1917 in Canada.
Meanwhile, on December 18, 1917, Herrer's attorney, Enrique, wrote to the Manila Office
stating that Herrer wanted to withdraw his application to which the office wrote a letter dated
November 26, 1917 stating that the policy had already been issued. The letter was received by
the attorney on December 21, 1917. Herrer had died a day earlier on December 20, 1920.

The trial court ruled that the contract had been perfected, hence this appeal.

ISSUES:

1. Whether or not the policyholder had received notice of the acceptance of his policy;
2. Whether or not the contract of life annuity was perfected.

HELD:

1. No. The facts clearly show that Herrer was not informed of the acceptance of the policy before
his death.

2. No. The contract was not perfected. Art. 1262 provides that acceptance by letter does not bind
the person making the offer except from the time it came to his knowledge. The pertinent fact is
that according to the provisional receipt, the insurance company had to:

1) conduct a medical examination;

2) had to obtain the head office's approval; and

3) somehow communicate such approval.

It is true that the letter notifying acceptance was deposited in the post office, but the fact of
notification is a rebuttable presumption and the facts clearly show that Herrer never received the
notice of the acceptance before his death.

 Great Pacific Life Assurance Co. vs. CA (G.R. Nos. 31845 & 31873, April 30, 1979)

FACTS:
Private respondent Ngo Hing filed a life insurance application with Great Pacific Life
Assurance (Pacific life) for a 20-year endowment policy in the amount of 50,000 on the life of
his 1 year old daughter Helen Go. Lapu-lapu Mondragon, the branch manager of Pacific life in
Cebu type-wrote the data on the application form which was signed by Ngo Hing. Not mentioned
in the application, the fact that Helen Go was a special child. Thereafter, Ngo Hing paid the
annual premium amounting to P1,077.00 but he retained the amount of P1317.00 as his
commission as a duly authorized agent of Pacific Life. Upon the payment of the premium, a
binding deposit receipt was issued to Ngo Hing. Also, at the back of the application form,
Mondragon wrote his strong recommendation for the application of Ngo Hing. Later, Mondragon
received a letter from Pacific life about its disapproval of the application because the life
insurance application of a 20-year endowment plan is not available for minors below 7 years of
age. In the same letter, Pacific life recommended that the Juvenile Triple Action Plan be instead
offered to Ngo Hing. The non-acceptance of the offer was not communicated by Mondragon to
Ngo Hing. Instead, he wrote back to Pacific Life pointing out that since 1954, the customers
especially the Chinese were asking for such coverage.
Eventually, Helen Go died of influenza with complications of bronchopneumonia. Consequently,
Ngo Hing sought the payment of the proceeds of the insurance, but having failed in his efforts,
he filed an action for recovery of the same before the CFI Cebu. CFI Cebu granted the said claim
of Ngo Hing. Hence, this case.

ISSUE/S:

1) Whether the binding deposit receipt constituted as a temporary contract of life


insurance.

2.) Whether Ngo Hing concealed the state of health and physical condition of Helen Go,
which rendered the binding deposit receipt as void.

RULING:

1) No, the Supreme Court ruled that, the binding deposit receipt in question is merely an
acknowledgment, on behalf of the company, that the latter's branch office had received from the
applicant the insurance premium and had accepted the application subject for processing by the
insurance company; and that the latter will either approve or reject the same on the basis of
whether or not the applicant is "insurable on standard rates." Since petitioner Pacific Life
disapproved the insurance application of respondent Ngo Hing, the binding deposit receipt in
question had never become in force at any time.

Upon this premise, the binding deposit receipt is, manifestly, merely conditional and does not
insure outright. As held by this Court, where an agreement is made between the applicant and the
agent, no liability shall attach until the principal approves the risk and a receipt is given by the
agent. The acceptance is merely conditional and is subordinated to the act of the company in
approving or rejecting the application.

It bears repeating that through the intra-company communication of April 30, 1957, Pacific Life
disapproved the insurance application in question on the ground that it is not offering the twenty-
year endowment insurance policy to children less than seven years of age. What it offered
instead is another plan known as the Juvenile Triple Action, which private respondent failed to
accept. In the absence of a meeting of the minds between petitioner Pacific Life and private
respondent Ngo Hing over the 20-year endowment life insurance in the amount of P50,000.00 in
favor of the latter's one-year old daughter, and with the non-compliance of the above-quoted
conditions stated in the disputed binding deposit receipt, there could have been no insurance
contract duly perfected between them. Accordingly, the deposit paid by private respondent shall
have to be refunded by Pacific Life.
A contract of insurance, like other contracts, must be assented to by both parties either in person
or by their agents ... The contract, to be binding from the date of the application, must have been
a completed contract, one that leaves nothing to be done, nothing to be completed, nothing to be
passed upon, or determined, before it shall take effect. There can be no contract of insurance
unless the minds of the parties have met in agreement.

2) Yes, the Supreme Court is of the firm belief that private respondent had deliberately concealed
the state of health and physical condition of his daughter Helen Go. When private respondent
supplied the required essential data for the insurance application form, he was fully aware that
his one-year old daughter is typically a mongoloid child. Such a congenital physical defect could
never be ensconced nor disguised. Nonetheless, private respondent, in apparent bad faith,
withheld the fact material to the risk to be assumed by the insurance company. As an insurance
agent of Pacific Life, he ought to know, as he surely must have known his duty and
responsibility to such a material fact. Had he diamond said significant fact in the insurance
application form Pacific Life would have verified the same and would have had no choice but to
disapprove the application outright.

The contract of insurance is one of perfect good faith uberrima fides meaning good faith,
absolute and perfect candor or openness and honesty; the absence of any concealment or
demotion, however slight, not for the alone but equally so for the insurer. Concealment is a
neglect to communicate that which a party knows and ought to communicate. Whether
intentional or unintentional the concealment entitles the insurer to rescind the contract of
insurance.

III. Insurable Interest

a. concept of insurable interest in general


b. reason for the requirement
c. insurable interest in life (own life, life of others, beneficiary)
 Philamcare Health System vs. CA, 379 SCRA 356

FACTS:

Respondent Julita Trinos’ deceased husband, Ernani Trinos applied for a health care coverage
with petitioner Philamcare Health Systems, Inc. In the standard application form, he answered
NO to the following question: Have you or any of your family members ever consulted or been
treated for high blood pressure, heart trouble, diabetes, cancer, liver disease, asthma or peptic
ulcer? The application was approved and extended for 13 months, until June 1, 1990. During the
period of the coverage, Ernani suffered a heart attack resulting in confinement for a month at the
Manila Medical Center (MMC). While her husband was in the hospital, respondent tried to claim
the benefits under the health care agreement. However, petitioner denied her claim saying that
the Health Care Agreement was void on the ground that there was a concealment regarding
Ernani’s medical history. After his discharge, Ernani was brought again at the Chinese General
Hospital where he died. Julita then filed an action for damages against Philamcare including its
President Dr. Benito Reverente. RTC ruled in favour of Julita, and this was affirmed by the CA
except that it deleted awards for damages and absolved Dr. Reverente.

ISSUE:

Whether or not the agreement is a contract of indemnity.

RULING:

Yes. The health care agreement was in the nature of non-life insurance, which is primarily a
contract of indemnity. The insurable interest of respondent’s husband in obtaining the health care
agreement was his own health. Once the member incurs hospital, medical or any other expense
arising from sickness, injury or other stipulated contingent, the health care provider must pay for
the same to the extent agreed upon under the contract. An insurance contract exists where the
following elements concur:

(a) The insured has an insurable interest;

(b) The insured is subject to a risk of loss by the happening of the designated peril;

(c) The insurer assumes the risk;

(d) Such assumption of risk is part of a general scheme to distribute actual losses among a large
group of persons bearing a similar risk; and

(e) In consideration of the insurer’s promise, the insured pays a premium.

 Lalican vs. Insular Life Assurance Co., Ltd., 597 SCRA 159

FACTS:
Violeta is the widow of Eulogio C. Lalican. Eulogio applied for an insurance policy with Insular
Life through its agent Malaluan. Policy No. 9011992 was issued to Eulogio, it contained a 20-
Year Endowment Variable Income Package Flexi Plan and Violeta was named as the primary
beneficiary.

Under the terms of the policy, Eulogio was to pay premiums on a quarterly basis every 24 April,
24 July, 24 October and 24 January of each year, until the end of the 20-year period of the
policy. There was a grace period of 31 days for the payment of each premium and if the premium
was not paid on the due date, then the policy would be in default, and if the premium remained
unpaid until the end of the grace period, the policy would become void.

Eulogio failed to pay the premium due on Jan 24 1998, even after the lapse of the grace period,
hence the period became void. Eulogio submitted to Insular Life an Application of Reinstatement
but it could not be fully processed because there was an overdue interest amounting to P322.

Eulogio went to Malaluan's house to submit a second Application for Reinstatement including
the payments. However on the same day Eulogio died of cardiac arrest.

Malaluan forwarded the application to Insular Life the next day but Insular Life no longer acted
upon the application as Eulogio had already passed away.

Violeta filed with the RTC a Complaint for Death Claim Benefit, the RTC rendered a Decision
in favor of Insular Life, hence this petition.

ISSUE:
Whether Eulogio was able to reinstate the lapsed insurance policy on his life before his death

RULING:
No. The policy had already lapsed. Eulogio's filing of an Application of Reinstatement
constitutes an admission that the policy had already lapsed.
Eulogio's death made it impossible to fully comply with the conditions of reinstatement in Policy
No. 9011992. The policy could only be considered reinstated after it had been processed and
approved by Insular Life during Eulogio's lifetime and good health. Only Insular Life had the
power to reinstate the policy, it does not matter that the application was with Malaluan before
Eulogio died.

 El Oriente Fabrica de Tabacos vs. Posada, 56 Phil. 147

Facts:
El Oriente, in order to protect itself against the loss that it might suffer by reason of the death of
its manager, Velhagen, who had had more than thirty-five (35) years of experience in the
manufacture of cigars in the Philippines, procured from the Manufacturers Life Insurance Co., of
Toronto, Canada an insurance policy on the life of the said Velhagen for the sum of $50,000,
United States currency designating itself as the beneficiary.

El Oriente paid for the premiums due thereon and charged them as expenses of its business, and
deducted the same from its gross income as reported in its annual ITR. Such deductions were
allowed upon showing that such premiums were legitimate expenses of its business.

Upon the death of Velhagen in 1929, El Oriente received all the proceeds of the said life
insurance policy, together with the interests and the dividends accruing thereon, aggregating
P104,957.88

CIR assessed El Oriente for deficiency taxes because El Oriente did not include as income the
proceeds received from the insurance.

Issue:
Whether or not the proceeds of insurance taken by a corporation on the life of an important
official to indemnify it against loss in case of his death, are taxable as income under the
Philippine Income Tax Law?

Ruling:
No, the proceeds of the insurance are not taxable.

In Chapter I of the Tax Code, is to be found Sec. 4 which provides that, "The following incomes
shall be exempt from the provisions of this law: (a) The proceeds of life insurance policies paid
to beneficiaries upon the death of the insured . . ."

Section 10, as amended, in Chapter II On Corporations, provides that, "There shall be levied,
assessed, collected, and paid annually upon the total net income received in the preceding
calendar year from all sources by every corporation . . .a tax of three per centum upon such
income . . ."
Section 11 in the same chapter, provides the exemptions under the law, but neither here nor in
any other section is reference made to the provisions of section 4 in Chapter I.

Under the view we take of the case, it is sufficient for our purposes to direct attention to the
anomalous and vague condition of the law. It is certain that the proceeds of life insurance
policies paid to individual beneficiaries upon the death of the insured are exempt. It is not so
certain that the proceeds of life insurance policies paid to corporate beneficiaries upon the death
of the insured are likewise exempt. But at least, it may be said that the law is indefinite in
phraseology and does not permit us unequivocally to hold that the proceeds of life insurance
policies received by corporations constitute income which is taxable.

d. insurable interest in property


 Spouses Nilo Cha and Stella Uy Cha vs. CA, GR 124520, August 18, 1997

FACTS:

Spouses Nilo Cha and Stella Uy Cha entered into a 1-year lease contract with private
respondent CKS Development Corporation. On their lease contract, one of its stipulations stated
that the lessee cannot insure against fire the merchandise placed in the leased premises unless a
written consent and approval from the lessor is obtained. Should the lessee fail to obtain such
written consent or approval, then the insurance policy will be deemed assigned and transferred to
the lessor for its own benefit. Despite such stipulation in their lease contract, Spouses Cha still
insured the merchandise inside the leased premises against loss by fire amounting to 500,000
pesos with United Insurance Co without the written consent of CKS. On the day that the lease
contract was about to expire, a fire broke out on the leased premises. CKS found out about the
insurance procured by Spouses Cha which led him to write the insurer a demand letter asking
that the proceeds of the insurance contract be paid directly to him based on the stipulation
indicated in his lease contract with Spouses Cha. United refused to pay CKS. Thus, the latter
filed a complaint against Spouses Cha and United. The RTC ruled in favor of CKS and ordered
United to pay CKS. Hence, this petition.

ISSUE:

Whether or not private respondent CKS Development Corporation can be the beneficiary
of a fire insurance policy taken by the lessee.
RULING:

No, CKS Development Corporation cannot be the beneficiary of a fire insurance policy
taken by the lessee.

Sec. 18 of the Insurance Code provides: “Sec. 18. No contract or policy of insurance on
property shall be enforceable except for the benefit of some person having an insurable interest
in the property insured.” A non-life insurance policy such as the fire insurance policy taken by
petitioner spouses over their merchandise is primarily a contract of indemnity. Insurable interest
in the property insured must exist at the time the insurance takes effect and at the time the loss
occurs. The basis of such requirement of insurable interest in property insured is based on sound
public policy: to prevent a person from taking out an insurance policy on property upon which he
has no insurable interest and collecting the proceeds of said policy in case of loss of the property.
In such a case, the contract of insurance is a mere wager which is void under Section 25 of the
Insurance Code.

Therefore, respondent CKS cannot, under the Insurance Code—a special law—be validly
a beneficiary of the fire insurance policy taken by the petitioner-spouses over their merchandise.
This insurable interest over said merchandise remains with the insured, the Cha spouses. The
automatic assignment of the policy to CKS under the provision of the lease contract previously
quoted is void for being contrary to law and/or public policy. The proceeds of the fire insurance
policy thus rightfully belong to the spouses Nilo Cha and Stella Uy-Cha (herein co-petitioners.)
The insurer (United) cannot be compelled to pay the proceeds of the fire insurance policy to a
person (CKS) who has no insurable interest in the property insured.

 Malayan Insurance Co. vs. PAP Co., GR 200784, August 7, 2013

FACTS:

On May 13, 1996, Malayan Insurance Company (Malayan) issued a Fire Insurance Policy
to PAP Co., Ltd. (PAP Co.) for the latter’s machineries and equipment located at Sanyo
Precision Phils. Bldg., Phase III, Lot 4, Block 15, PEZA, Rosario, Cavite (Sanyo Building). The
insurance, which was for Fifteen Million Pesos (₱15,000,000.00) and effective for a period of
one (1) year, was procured by PAP Co. for Rizal Commercial Banking Corporation (RCBC), the
mortgagee of the insured machineries and equipment. After the passage of almost a year but
prior to the expiration of the insurance coverage, PAP Co. renewed the policy on an “as is” basis.
Pursuant thereto, a renewal policy was issued by Malayan to PAP Co. for the period May 13,
1997 to May 13, 1998. On October 12, 1997 and during the subsistence of the renewal policy,
the insured machineries and equipment were totally lost by fire. Hence, PAP Co. filed a fire
insurance claim with Malayan in the amount insured. In a letter, dated December 15, 1997,
Malayan denied the claim upon the ground that, at the time of the loss, the insured machineries
and equipment were transferred by PAP Co. to a location different from that indicated in the
policy. Specifically, that the insured machineries were transferred in September 1996 from the
Sanyo Building to the Pace Pacific Bldg., Lot 14, Block 14, Phase III, PEZA, Rosario, Cavite
(Pace Pacific). Contesting the denial, PAP Co. argued that Malayan cannot avoid liability as it
was informed of the transfer by RCBC, the party duty-bound to relay such information.
However, Malayan reiterated its denial of PAP Co.’s claim. Distraught, PAP Co. filed the
complaint below against Malayan.

ISSUE:

Whether or not Malayan should be held liable under the fire insurance policy

RULING:

NO. The Court agrees with the position of Malayan that it cannot be held liable for the
loss of the insured properties under the fire insurance policy.

The policy forbade the removal of the insured properties unless sanctioned by Malayan.
Evidently, by the clear and express condition in the renewal policy, the removal of the insured
property to any building or place required the consent of Malayan. Any transfer effected by the
insured, without the insurer’s consent, would free the latter from any liability. The transfer from
the Sanyo Factory to the PACE Factory increased the risk. The Court agrees with Malayan that
the transfer to the Pace Factory exposed the properties to a hazardous environment and
negatively affected the fire rating stated in the renewal policy. The increase in tariff rate from
0.449% to 0.657% put the subject properties at a greater risk of loss. Such increase in risk would
necessarily entail an increase in the premium payment on the fire policy. Unfortunately, PAP
chose to remain completely silent on this very crucial point. Despite the importance of the issue,
PAP failed to refute Malayan’s argument on the increased risk. Malayan is entitled to rescind the
insurance contract. Considering that the original policy was renewed on an “as is basis,” it
follows that the renewal policy carried with it the same stipulations and limitations. The terms
and conditions in the renewal policy provided, among others, that the location of the risk insured
against is at the Sanyo factory in PEZA. The subject insured properties, however, were totally
burned at the Pace Factory. Although it was also located in PEZA, Pace Factory was not the
location stipulated in the renewal policy. There being an unconsented removal, the transfer was
at PAP’s own risk. Consequently, it must suffer the consequences of the fire. It can also be said
that with the transfer of the location of the subject properties, without notice and without
Malayan’s consent, after the renewal of the policy, PAP clearly committed concealment,
misrepresentation and a breach of a material warranty. Moreover, under Section 168 of the
Insurance Code, the insurer is entitled to rescind the insurance contract in case of an alteration in
the use or condition of the thing insured.

Accordingly, an insurer can exercise its right to rescind an insurance contract when the
following conditions are present, to wit:

1) The policy limits the use or condition of the thing insured;

2) There is an alteration in said use or condition;

3) The alteration is without the consent of the insurer;

4) The alteration is made by means within the insured's control; and

5) The alteration increases the risk of loss

In the case at bench, all these circumstances are present. It was clearly established that
the renewal policy stipulated that the insured properties were located at the Sanyo factory; that
PAP removed the properties without the consent of Malayan; and that the alteration of the
location increased the risk of loss.

e. change of interest in the thing insured

 Spouses Cha vs. CA, (August 18, 1997)

FACTS:

Spouses Nilo Cha and Stella Uy Cha entered into a 1-year lease contract with private
respondent CKS Development Corporation. On their lease contract, one of its stipulations stated
that the lessee cannot insure against fire the merchandise placed in the leased premises unless a
written consent and approval from the lessor is obtained. Should the lessee fail to obtain such
written consent or approval, then the insurance policy will be deemed assigned and transferred to
the lessor for its own benefit. Despite such stipulation in their lease contract, Spouses Cha still
insured the merchandise inside the leased premises against loss by fire amounting to 500,000
pesos with United Insurance Co without the written consent of CKS. On the day that the lease
contract was about to expire, a fire broke out on the leased premises. CKS found out about the
insurance procured by Spouses Cha which led him to write the insurer a demand letter asking
that the proceeds of the insurance contract be paid directly to him based on the stipulation
indicated in his lease contract with Spouses Cha. United refused to pay CKS. Thus, the latter
filed a complaint against Spouses Cha and United. The RTC ruled in favor of CKS and ordered
United to pay CKS. Hence, this petition.

ISSUE:

Whether or not private respondent CKS Development Corporation can be the beneficiary
of a fire insurance policy taken by the lessee.

RULING:

No, CKS Development Corporation cannot be the beneficiary of a fire insurance policy
taken by the lessee.

Sec. 18 of the Insurance Code provides: “Sec. 18. No contract or policy of insurance on
property shall be enforceable except for the benefit of some person having an insurable interest
in the property insured.” A non-life insurance policy such as the fire insurance policy taken by
petitioner spouses over their merchandise is primarily a contract of indemnity. Insurable interest
in the property insured must exist at the time the insurance takes effect and at the time the loss
occurs. The basis of such requirement of insurable interest in property insured is based on sound
public policy: to prevent a person from taking out an insurance policy on property upon which he
has no insurable interest and collecting the proceeds of said policy in case of loss of the property.
In such a case, the contract of insurance is a mere wager which is void under Section 25 of the
Insurance Code.

Therefore, respondent CKS cannot, under the Insurance Code—a special law—be validly
a beneficiary of the fire insurance policy taken by the petitioner-spouses over their merchandise.
This insurable interest over said merchandise remains with the insured, the Cha spouses. The
automatic assignment of the policy to CKS under the provision of the lease contract previously
quoted is void for being contrary to law and/or public policy. The proceeds of the fire insurance
policy thus rightfully belong to the spouses Nilo Cha and Stella Uy-Cha (herein co-petitioners.)
The insurer (United) cannot be compelled to pay the proceeds of the fire insurance policy to a
person (CKS) who has no insurable interest in the property insured.

 Geagonia vs. CA, February 6, 1995)

FACTS:
The petitioner is the owner of Norman’s Mart located in the public market of San Francisco,
Agusan del Sur. On 22 December 1989, he obtained from the private respondent fire insurance
policy No. F-14622 2 for P100,000.00. The period of the policy was from 22 December 1989 to
22 December 1990 and covered the following: "Stock-in-trade consisting principally of dry
goods such as RTW’s for men and women wear and other usual to assured’s business." The
petitioner declared in the policy under the subheading entitled CO-INSURANCE that Mercantile
Insurance Co., Inc. was the co-insurer for P50,000.00. From 1989 to 1990, the petitioner had in
his inventory stocks amounting to P392,130.50.

The policy contained the following condition:


"3. The insured shall give notice to the Company of any insurance or insurances already effected,
or which may subsequently be effected, covering any of the property or properties consisting of
stocks in trade, goods in process and/or inventories only hereby insured, and unless notice be
given and the particulars of such insurance or insurances be stated therein or endorsed in this
policy pursuant to Section 50 of the Insurance Code, by or on behalf of the Company before the
occurrence of any loss or damage, all benefits under this policy shall be deemed forfeited,
provided however, that this condition shall not apply when the total insurance or insurances in
force at the time of the loss or damage is not more than P200,000.00."

The petitioners’ stocks were destroyed by fire. He then filed a claim which was subsequently
denied because the petitioner’s stocks were covered by two other fire insurance policies for Php
200,000 issued by Cebu Branch of the Philippines First Insurance Co., Inc. (PFIC). The basis of
the private respondent's denial was the petitioner's alleged violation of Condition 3 of the policy.

Geagonia then filed a complaint against the private respondent in the Insurance Commission for
the recovery of P100,000.00 under fire insurance policy and damages. He claimed that he knew
the existence of the other two policies. But, he said that he had no knowledge of the provision in
the private respondent's policy requiring him to inform it of the prior policies and this
requirement was not mentioned to him by the private respondent's agent. The Insurance
Commission found that the petitioner did not violate Condition 3 as he had no knowledge of the
existence of the two fire insurance policies obtained from the PFIC; that it was Cebu Tesing
Textiles which procured the PFIC policies without informing him or securing his consent; and
that Cebu Tesing Textile, as his creditor, had insurable interest on the stocks. The Insurance
Commission then ordered the respondent company to pay the complainant the sum of
P100,000.00 with interest and attorney’s fees.
CA reversed the decision of the Insurance Commission because it found that the petitioner knew
of the existence of the two other policies issued by the PFIC.

ISSUE/S:
1. WON the petitioner had not disclosed the two insurance policies when he obtained the fire
insurance and thereby violated Condition 3 of the policy.
2. WON he is prohibited from recovering.

RULING:
1. We agree with the Court of Appeals that the petitioner knew of the prior policies issued
by the PFIC. His letter of 18 January 1991 to the private respondent conclusively proves
this knowledge. His testimony to the contrary before the Insurance Commissioner and
which the latter relied upon cannot prevail over a written admission made ante litem
motam. It was, indeed, incredible that he did not know about the prior policies since these
policies were not new or original. In fact the policies were renewed twice.
2. Stated differently, provisions, conditions or exceptions in policies which tend to work a
forfeiture of insurance policies should be construed most strictly against those for whose
benefits they are inserted, and most favorably toward those against whom they are
intended to operate. With these principles in mind, Condition 3 of the subject policy is
not totally free from ambiguity and must be meticulously analyzed. Such analysis leads
us to conclude that (a) the prohibition applies only to double insurance, and (b) the nullity
of the policy shall only be to the extent exceeding P200,000.00 of the total policies
obtained. Furthermore, by stating within Condition 3 itself that such condition shall not
apply if the total insurance in force at the time of loss does not exceed P200,000.00, the
private respondent was amenable to assume a co-insurer's liability up to a loss not
exceeding P200,000.00. What it had in mind was to discourage over-insurance. Indeed,
the rationale behind the incorporation of "other insurance" clauses in fire policies is to
prevent over-insurance and thus avert the perpetration of fraud. When a property owner
obtains insurance policies from two or more insurers in a total amount that exceeds the
property's value, the insured may have an inducement to destroy the property for the
purpose of collecting the insurance. The public as well as the insurer is interested in
preventing a situation in which a fire would be profitable to the insured.

 RCBC vs. CA (289 SCRA 292)


 Gaisano Cagayan, Inc. vs. Insurance Company of North America, G.R. No. 147839 (June 8,
2006).
FACTS:

Intercapitol Marketing Corporation (IMC) is the maker of Wrangler Blue Jeans. while
Levi Strauss (Phils.) Inc. (LSPI) is the local distributor of products bearing trademarks owned by
Levi Strauss & Co.

IMC and LSPI separately obtained from Insurance Company of North America fire
insurance policies for their book debt endorsements related to their ready-made clothing
materials which have been sold or delivered to various customers and dealers of the Insured
anywhere in the Philippines which are unpaid 45 days after the time of the loss.

On February 25, 1991, Gaisano Superstore Complex in Cagayan de Oro City, owned by
Gaisano Cagayan, Inc., containing the ready-made clothing materials sold and delivered by IMC
and LSPI was consumed by fire.

On February 4, 1992, the Insurance Company of North America filed a complaint for
damages against Gaisano Cagayan, Inc. alleges that IMC and LSPI filed their claims under their
respective fire insurance policies which it paid thus it was subrogated to their rights.

In its counter claim, Gaisano Cagayan, Inc contends that it cannot be held liable because
it was destroyed due to fortuities event or force majeure.

The RTC dismissed respondent’s complaint. It held that fire was not attributable to the
negligence of the petitioner and that IMC and LSPI retained ownership of the delivered goods
until fully paid, it must bear the loss (res perit domino).

Dissatisfied, petitioner appealed to the CA who rendered its decision setting aside the
decision of the RTC.

ISSUE/S:

Whether or not there is an insurable interest on book debts.

RULING:

YES. Section 13 of our Insurance Code defines insurable interest as "every interest in
property, whether real or personal, or any relation thereto, or liability in respect thereof, of such
nature that a contemplated peril might directly damnify the insured." Parenthetically, under
Section 14 of the same Code, an insurable interest in property may consist in:

(a) an existing interest;

(b) an inchoate interest founded on existing interest; or

(c) an expectancy, coupled with an existing interest in that out of which the expectancy arises.
Therefore, an insurable interest in property does not necessarily imply a property interest
in, or a lien upon, or possession of, the subject matter of the insurance, and neither the title nor a
beneficial interest is requisite to the existence of such an interest, it is sufficient that the insured
is so situated with reference to the property that he would be liable to loss should it be injured or
destroyed by the peril against which it is insured. Anyone has an insurable interest in property
who derives a benefit from its existence or would suffer loss from its destruction. Indeed, a
vendor or seller retains an insurable interest in the property sold so long as he has any interest
therein, in other words, so long as he would suffer by its destruction, as where he has a vendor's
lien. In this case, the insurable interest of IMC and LSPI pertain to the unpaid accounts appearing
in their Books of Account 45 days after the time of the loss covered by the policies, and the
petitioner is liable for such unpaid accounts.

IV. Devices for ascertaining and controlling risk and loss

a. concealment
1. concept
2. duty to communicate
3. test of materiality
4. effect of concealment
5. matters which need not be communicated
6. waiver of information
 Qua Chee Gan vs. Law Union & Rock Insurance Co., 98 Phil. 85
 Great Pacific Life vs. CA, GR L-31845, April 30, 1979

FACTS:
Private respondent Ngo Hing filed a life insurance application with Great Pacific Life
Assurance (Pacific life) for a 20-year endowment policy in the amount of 50,000 on the life of
his 1 year old daughter Helen Go. Lapu-lapu Mondragon, the branch manager of Pacific life in
Cebu type-wrote the data on the application form which was signed by Ngo Hing. Not mentioned
in the application, the fact that Helen Go was a special child. Thereafter, Ngo Hing paid the
annual premium amounting to P1,077.00 but he retained the amount of P1317.00 as his
commission as a duly authorized agent of Pacific Life. Upon the payment of the premium, a
binding deposit receipt was issued to Ngo Hing. Also, at the back of the application form,
Mondragon wrote his strong recommendation for the application of Ngo Hing. Later, Mondragon
received a letter from Pacific life about its disapproval of the application because the life
insurance application of a 20-year endowment plan is not available for minors below 7 years of
age. In the same letter, Pacific life recommended that the Juvenile Triple Action Plan be instead
offered to Ngo Hing. The non-acceptance of the offer was not communicated by Mondragon to
Ngo Hing. Instead, he wrote back to Pacific Life pointing out that since 1954, the customers
especially the Chinese were asking for such coverage.

Eventually, Helen Go died of influenza with complications of bronchopneumonia. Consequently,


Ngo Hing sought the payment of the proceeds of the insurance, but having failed in his efforts,
he filed an action for recovery of the same before the CFI Cebu. CFI Cebu granted the said claim
of Ngo Hing. Hence, this case.

ISSUE/S:

1) Whether the binding deposit receipt constituted as a temporary contract of life


insurance.

2.) Whether Ngo Hing concealed the state of health and physical condition of Helen Go,
which rendered the binding deposit receipt as void.

RULING:

1) No, the Supreme Court ruled that, the binding deposit receipt in question is merely an
acknowledgment, on behalf of the company, that the latter's branch office had received from the
applicant the insurance premium and had accepted the application subject for processing by the
insurance company; and that the latter will either approve or reject the same on the basis of
whether or not the applicant is "insurable on standard rates." Since petitioner Pacific Life
disapproved the insurance application of respondent Ngo Hing, the binding deposit receipt in
question had never become in force at any time.

Upon this premise, the binding deposit receipt is, manifestly, merely conditional and does not
insure outright. As held by this Court, where an agreement is made between the applicant and the
agent, no liability shall attach until the principal approves the risk and a receipt is given by the
agent. The acceptance is merely conditional and is subordinated to the act of the company in
approving or rejecting the application.

It bears repeating that through the intra-company communication of April 30, 1957, Pacific Life
disapproved the insurance application in question on the ground that it is not offering the twenty-
year endowment insurance policy to children less than seven years of age. What it offered
instead is another plan known as the Juvenile Triple Action, which private respondent failed to
accept. In the absence of a meeting of the minds between petitioner Pacific Life and private
respondent Ngo Hing over the 20-year endowment life insurance in the amount of P50,000.00 in
favor of the latter's one-year old daughter, and with the non-compliance of the above-quoted
conditions stated in the disputed binding deposit receipt, there could have been no insurance
contract duly perfected between them. Accordingly, the deposit paid by private respondent shall
have to be refunded by Pacific Life.
A contract of insurance, like other contracts, must be assented to by both parties either in person
or by their agents ... The contract, to be binding from the date of the application, must have been
a completed contract, one that leaves nothing to be done, nothing to be completed, nothing to be
passed upon, or determined, before it shall take effect. There can be no contract of insurance
unless the minds of the parties have met in agreement.

2) Yes, the Supreme Court is of the firm belief that private respondent had deliberately concealed
the state of health and physical condition of his daughter Helen Go. When private respondent
supplied the required essential data for the insurance application form, he was fully aware that
his one-year old daughter is typically a mongoloid child. Such a congenital physical defect could
never be ensconced nor disguised. Nonetheless, private respondent, in apparent bad faith,
withheld the fact material to the risk to be assumed by the insurance company. As an insurance
agent of Pacific Life, he ought to know, as he surely must have known his duty and
responsibility to such a material fact. Had he diamond said significant fact in the insurance
application form Pacific Life would have verified the same and would have had no choice but to
disapprove the application outright.

The contract of insurance is one of perfect good faith uberrima fides meaning good faith,
absolute and perfect candor or openness and honesty; the absence of any concealment or
demotion, however slight, not for the alone but equally so for the insurer. Concealment is a
neglect to communicate that which a party knows and ought to communicate. Whether
intentional or unintentional the concealment entitles the insurer to rescind the contract of
insurance.

 Ng Gan Zee vs. Asian Crusader Life, GR L-30685, May 30, 1983

Facts:

On May 12, 1962, Kwong Nam applied for a 20-year endowment insurance on his life for
the sum of P20,000.00, with his wife, appellee Ng Gan Zee, as beneficiary. On the same
date, appellant, upon receipt of the required premium from the insured, approved the
application and issued the corresponding policy. On December 6, 1963, Kwong Nam died
of cancer of the liver with metastasis. All premiums had been religiously paid at the time of
his death. His widow Ng Gan Zee attempted to claim the payment of the face value of the
policy and also submitted the required proof of death of the insured. Appellant denied the
claim on the ground that the answers given by the insured to the questions appearing in his
application for life insurance were untrue. Appellee brought the matter to the attention of
the Insurance Commissioner who said that was no material concealment on the part of the
insured and that appellee should be paid the full-face value of the policy. Despite this,
appellant refused to settle its obligation.
Appellant alleged that the insured was guilty of misrepresentation when he answered “No”
to this question: “Has any life insurance company ever refused your application for
insurance or for reinstatement of a lapsed policy or offered you a policy different from that
applied for? If, so, name company and date.” The lower court found the argument bereft of
factual basis. Appellant further maintains that when the insured was examined in
connection with his application for life insurance, he gave the appellant’s medical examiner
false and misleading information as to his ailment and previous operation. Using the
medical data showing that the insured was operated on for “peptic ulcer”, involving the
excision of a portion of the stomach, appellant argues that the insured’s statement in his
application that a tumor associated with ulcer of the stomach, “hard and of a hen’s egg
size,” was removed during said operation, constituted material concealment.

Issue:

Whether or not the appellant, because of insured’s aforesaid representation, was misled or
deceived into entering the contract or in accepting the risk at the rate of premium agreed
upon.

Ruling:

No. Concealment exists where the assured had knowledge of a fact material to the risk, and
honesty, good faith, and fair dealing requires that he should communicate it to the assurer
but he designedly and intentionally withholds the same. The concealment must, in the
absence of inquiries, be not only material, but fraudulent, or the fact must have been
intentionally withheld in order for the insurer to rescind the contract. Misrepresentation as
a defense of the insurer to avoid liability is an ‘affirmative’ defense. The duty to establish
such a defense by satisfactory and convincing evidence rests upon the defendant. The
evidence before the Court does not clearly and satisfactorily establish that defense.

Kwong Nam had informed the appellant’s medical examiner that the tumor for which he
was operated on was “associated with ulcer of the stomach.” In the absence of evidence that
the insured had sufficient medical knowledge as to enable him to distinguish between
“peptic ulcer” and “a tumor”, his statement that said tumor was “associated with ulcer of
the stomach,” should be construed as an expression made in good faith of his belief as to
the nature of his ailment and operation.

Even if the information given was imperfect, the same was nevertheless sufficient to have
induced appellant to make further inquiries about the ailment and operation of the
insured. It has been held that where, “upon the face of the application, a question appears
to be not answered at all or to be imperfectly answered, and the insurers issue a policy
without any further inquiry, they waive the imperfection of the answer and render the
omission to answer more fully immaterial. (sec. 32 Insurance Law). The defendant was too
eager to accept the application and receive the insured’s premium. It would be inequitable
now to allow the defendant to avoid liability under the circumstances. Judgment appealed
from is AFFIRMED.

 New Life Enterprises and Julian Sy vs. CA, GR 94071, March 31, 1992

Facts: Julian Sy and Jose Sy Bang have formed a business partnership in the City of Lucena.
Under the business name of New Life Enterprises, the partnership engaged in the sale of
construction materials at its place of business, a two storey building situated at Iyam, Lucena
City. The facts show that Julian Sy insured the stocks in trade of New Life Enterprises with
Western Guaranty Corporation (P350,000.00), Reliance Surety and Insurance. Co., Inc.
(P1,000,000.00), and Equitable Insurance Corporation (P200,000.00). Thus, when the building
occupied by the New Life Enterprises was gutted by fire at about 2:00 o'clock in the morning of
October 19, 1982, the stocks in the trade inside said building were insured against fire in the total
amount of P1,550,000.00. The cause of fire was electrical in nature. According to the plaintiffs,
the building and the stocks inside were burned. After the fire, Julian Sy went to the insurance
companies and claimed for the insurance indemnities but these were rejected for violation of
policy conditions. Because of the denial of their claims for payment by the three (3) insurance
companies, petitioner filed separate civil actions against the former before the Regional Trial
Court of Lucena City. The RTC rules in favor of the respondent. Court of Appeals reversed said
judgment of the trial court, hence this petition.

Issue:

Whether or not Conditions Nos. 3 and 27 of the insurance contracts were violated by petitioners
thereby resulting in their forfeiture of all the benefits thereunder.

Ruling:

Condition No. 3 of said insurance policy provides that the insured shall give notice to the
Company of any insurance or insurances already effected, or which may subsequently be
effected, covering any of the property or properties consisting of stocks in trade, goods in
process and/or inventories only hereby insured, and unless such notice be given and the
particulars of such insurance or insurances be stated therein or endorsed on this policy pursuant
to Section 50 of the Insurance Code, by or on behalf of the Company before the occurrence of
any loss or damage, all benefits under this policy shall be deemed forfeited, provided however,
that this condition shall not apply when the total insurance or insurances in force at the time of
loss or damage not more than P200,000.00.
The terms of the contract are clear and unambiguous. The insured is specifically required to
disclose to the insurer any other insurance and its particulars which he may have effected on the
same subject matter. The knowledge of such insurance by the insurer's agents, even assuming the
acquisition thereof by the former, is not the "notice" that would estop the insurers from denying
the claim. Besides, the so-called theory of imputed knowledge, that is, knowledge of the agent is
knowledge of the principal, aside from being of dubious applicability here has likewise been
roundly refuted by respondent court whose factual findings we find acceptable.

Furthermore, when the words and language of documents are clear and plain or readily
understandable by an ordinary reader thereof, there is absolutely no room for interpretation or
construction anymore. Courts are not allowed to make contracts for the parties; rather, they will
intervene only when the terms of the policy are ambiguous, equivocal, or uncertain. The parties
must abide by the terms of the contract because such terms constitute the measure of the insurer's
liability and compliance therewith is a condition precedent to the insured's right of recovery from
the insurer.

While it is a cardinal principle of insurance law that a policy or contract of insurance is to be


construed liberally in favor of the insured and strictly against the insurer company, yet contracts
of insurance, like other contracts, are to be construed according to the sense and meaning of the
terms which the parties themselves have used. If such terms are clear and unambiguous, they
must be taken and understood in their plain, ordinary and popular sense. Moreover, obligations
arising from contracts have the force of law between the contracting parties and should be
complied with in good faith.

 Sunlife Assurance Co. of Canada vs. CA, GR 105135, June 22, 1995

FACTS:

On 15 April 1986, Robert John Bacani (RJ for brevity) procured a life insurance contract from
Sunlife Assurance Co. of Canada, designating his mother, private respondent Bernarda Bacani as
the beneficiary. On 26 June 1987, RJ died in a plane crash prompting respondent to file a claim,
seeking the benefits of the insurance policy. Nonetheless, in an investigation conducted by
Sunlife, it was found out that RJ did not disclose material facts relevant to the issuance of the
policy, rendering the insurance contract voidable. RJ asserted that he had no past medical
conditions, but the contrary was found out by the investigation. In fact, 2 weeks prior to RJ’s
application for insurance, he was diagnosed for renal failure and was confined in the Lung
Center of the Philippines.

Private respondent filed an action for specific performance before the trial court, which
eventually ruled in their favor. The lower court ruled that the facts concealed by the RJ were
made in good faith and under a belief that they need not be disclosed. Moreover, it held that the
health history of RJ was immaterial since the insurance policy was “non-medical.” The Court of
Appeals affirmed said decision.

ISSUE/S:

Whether the concealment of such fact, despite it not being the cause of death of RJ, is sufficient
to render the insurance contract voidable.

RULING:

The Court ruled in the positive.

Concealment is a neglect in communicating that which a party knows and ought to communicate.
Sec. 26 of the Insurance Code requires a party to communicate to the other, in good faith, all
facts within his knowledge which are material to the contract and as to which he makes no
warranty, and which the other has no means of ascertaining. Materiality is to be determined not
by the event, but solely by the probable and reasonable influence of the facts upon the party to
whom communication is due, in forming his estimate of the disadvantages of the proposed
contract or in making his inquiries.

The terms of the contract are clear, which required RJ to disclose to the Sunlife matters relating
to his health. The information which RJ failed to disclose were material and relevant to the
approval and issuance of the insurance policy. The matters concealed would have definitely
affected petitioner’s action on his application, either by approving it with the corresponding
adjustment for a higher premium or rejecting the same. Moreover, a disclosure may have
warranted a medical examination of RJ by Sunlife for it to reasonably assess the risk involved in
accepting the application.

Good faith is no defense in concealment. RJ’s failure to disclose the fact that he was
hospitalized for 2 weeks prior to filing his application for insurance raises grave doubts about his
bona fides.

Anent the finding that the facts concealed had no bearing to the cause of death of RJ, it is well
settled in Henson v The Philippines American Life Insurance Co. that the insured need not die of
the disease he had failed to disclose to the insurer. It is sufficient that his non-disclosure misled
the insurer in forming his estimates of the risks of the proposed insurance policy or in making
inquiries.

 Saturnino vs. Philamlife, 7 SCRA 316


Facts:

The policy sued upon is one for 20-year endowment non-medical insurance. This kind of
policy dispenses with the medical examination of the applicant usually required in ordinary life
policies. However, detailed information is called for in the application concerning the applicant’s
health and medical history. The written application in this case was submitted by Saturnino to
appellee on November 16, 1957, witnessed by appellee’s agent Edward A. Santos. The policy
was issued on the same day, upon payment of the first year’s premium of P339.25.

On September 19, 1958 Saturnino died of pneumonia, secondary to influenza. Appellants


here, who are her surviving husband and minor child, respectively, demanded payment of the
face value of the policy. The claim was rejected and this suit was subsequently instituted. It
appears that two months prior to the issuance of the policy or on September 9, 1957, Saturnino
was operated on for cancer, involving complete removal of the right breast, including the
pectoral muscles and the glands found in the right armpit. She stayed in the hospital for a period
of eight days, after which she was discharged, although according to the surgeon who operated
on her she could not be considered definitely cured, her ailment being of the malignant type.
Notwithstanding the fact of her operation Estefania A. Saturnino did not make a disclosure
thereof in her application for insurance. On the contrary, she stated therein that she did not have,
nor had she ever had, among other ailments listed in the application, cancer or other tumors; that
she had not consulted any physician, undergone any operation or suffered any injury within the
preceding five years; and that she had never been treated for nor did she ever have any illness or
disease peculiar to her sex, particularly of the breast, ovaries, uterus, and menstrual disorders.
The application also recites that the foregoing declarations constituted “a further basis for the
issuance of the policy.”

Issue:

Whether or not the failure of Saturnino to disclose the severity of her previous illness is material
to the avoidance of the insurance policy.

Ruling:

Yes. In the application for insurance signed by the insured in this case, she agreed to
submit to a medical examination by a duly appointed examiner of appellee if in the latter’s
opinion such examination was necessary as further evidence of insurability. In not asking her to
submit to a medical examination, appellants maintain, appellee was guilty of negligence, which
precluded it from finding about her actual state of health. No such negligence can be imputed to
appellee. It was precisely because the insured had given herself a clean bill of health that
appellee no longer considered an actual medical checkup necessary.
In the first place the concealment of the fact of the operation itself was fraudulent, as
there could not have been any mistake about it, no matter what the ailment. Secondly, in order to
avoid a policy it is not necessary to show actual fraud on the part of the insured.

In this jurisdiction a concealment, whether intentional or unintentional, entitles the


insurer to rescind the contract of insurance, concealment being defined as “negligence to
communicate that which a party knows and ought to communicate” (Sections 24 & 26, Act No.
2427). In the case of Argente v. West Coast Life Insurance Co., 51 Phil. 725, 732, this Court
said, quoting from Joyce, The Law of Insurance, 2nd ed., Vol. 3:

“The basis of the rule vitiating the contract in cases of concealment is that it misleads or
deceives the insurer into accepting the risk, or accepting it at the rate of premium agreed upon.
The insurer, relying upon the belief that the assured will disclose every material fact within his
actual or presumed knowledge, is misled into a belief that the circumstance withheld does not
exist, and he is thereby induced to estimate the risk upon a false basis that it does not exist.”

 Thelma vda. De Canilang vs. CA and Grepalife, 223 SCRA 443

FACTS:

Jaime Canilang consulted Dr. Wilfredo B. Claudio and was diagnosed as suffering from "sinus
tachycardia” and then "acute bronchitis." On next day, Jaime Canilang applied for a "non-
medical" insurance policy with respondent Great Pacific Life Assurance Company naming his
wife, Thelma Canilang, as his beneficiary. Jaime Canilang died of "congestive heart failure,"
"anemia," and "chronic anemia." Thelma, widow and beneficiary filed a claim with Great Pacific
which the insurer denied upon the ground that the insured had concealed material information
from it. Great Pacific presented Dr. Esperanza Quismorio, a physician working for Great Pacific.
She testified that the deceased's insurance application had been approved on the basis of his
medical declaration.

Insurance Commissioner Ansaldo ruled in favor of Vda. De Canilang holding that the failure to
convey certain information to the insurer was not "intentional" in nature. That Jaime Canilang
believed that he was suffering from minor ailment like a common cold. On appeal by Great
Pacific, the Court of Appeals reversed and set aside the decision of the Insurance Commissioner.
The Court of Appealed found that Jaime Canilang made a material concealment as the state of
his health at the time of the filing of insurance application, justifying respondent's denial of the
claim. That the failure of Jaime Canilang to disclose previous medical consultation and treatment
constituted material information which should have been communicated to Great Pacific to
enable the latter to make proper inquiries.

ISSUES:

(1) Whether or not Jaime Canilang concealed material information warranting denial of
insurance claim

(2) Whether or not unintentional nature of failure to convey information should be given
due course

RULING:

(1)

Yes. In the Medical Declaration made by Jaime Canilang, he set out that he has not been
confined in any hospital, sanitarium or infirmary, nor receive any medical or surgical
advice/attention within the last 5 years and have never been treated nor consulted a physician for
a heart condition, high blood pressure, cancer, diabetes, lung, kidney, stomach disorder, or any
other physical impairment. He failed to disclose that he had twice consulted Dr. Wilfredo B.
Claudio who had found him to be suffering from "sinus tachycardia" and "acute bronchitis."

Insurance Code of 1978, Sec. 26 provides that “a neglect to communicate that which a party
knows and ought to communicate, is called a concealment”. The information concealed must be
information which the concealing party knew and "ought to have communicated" information
which was "material to the contract." The test of materiality is contained in Section 31 of the
Insurance Code of 1978 which reads “materiality is to be determined not by the event, but solely
by the probable and reasonable influence of the facts upon the party to whom the communication
is due, in forming his estimate of the disadvantages of the proposed contract, or in making his
inquiries”

The information which Jaime Canilang failed to disclose was material to the ability of Great
Pacific to estimate the probable risk he presented as a subject of life insurance. Had Canilang
disclosed his visits to his doctor, the diagnosis made and medicines prescribed by such doctor, in
the insurance application, it may be reasonably assumed that Great Pacific would have made
further inquiries and would have probably refused to issue a non-medical insurance policy or, at
the very least, required a higher premium for the same coverage.

(2)

Sec. 27 of the Insurance Code of 1978 as amended by B.P. Blg. 874 provides that “a
concealment whether intentional or unintentional entitles the injured party to rescind a contract
of insurance”. Section 27 of the Insurance Code of 1978 is properly read as referring to "any
concealment" without regard to whether such concealment is intentional or unintentional.
 The Insular Life Assurance Co., Ltd. vs. Heirs of Alvarez, GR 207526 & 210156, October 3,
2018

FACTS:

Jose H. Alvarez and his wife owned a residential lot in Caloocan. In 1997, Union Bank
approved the housing loan of Alvarez in the amount of P648,000. A promissory note and real
estate secured the loan. Jose Alvarez was included among the mortgagors in the list of qualified
debtors covered by the Group Mortgage Redemption Insurance that Union Bank had with Insular
Life. A Mortgage Redemption Insurance was taken in the life of Alvarez. Jose Alvarez died in
April 17, 1998.
Union Bank filed with Insular Life a death claim under Alvarez’s name pursuant to the
Group Mortgage Redemption Insurance. Insular Life denied the claim because Alvarez did not
qualify for the coverage. He was supposedly more than 60 years when he applied for the loan.
The monthly amortizations of Alvarez stood unpaid. The lot was foreclosed and it was sold in
public auction having Union Bank as the highest bidder.
In 2001, the heirs of Alvarez filed a complaint for Declaration of the Nullity of Contract
and Damages against Union Bank, Alfonso Miranda (supposedly benefitted from the loan), and
the insurer identified only as John Doe. The heirs denied having any knowledge of the loan
obtained by Alvarez. The complaint was amended and became specific performance including
demand against Insular Life to fulfill its obligation as insurer under Group Mortgage Redemption
Insurance.
Union Bank argued against the ignorance raised by the heirs because of the Special
Power of Attorney executed by Adelina in favor of her husband.
The RTC ruled in favor of the heirs of Alvarez. It ruled that Alvarez had no fraudulent
intent when he gave Union Bank information about his age and date of birth. According to RTC,
Union Bank negotiated the Group Mortgage Insurance with Insular Life and ordinary customers
will not know about insurance policies unless it is brought to their knowledge by the bank. Also,
Union Bank had been in possession of the materials sufficient to inform itself of Alvarez
personal circumstances.
Insular Life argued for concealment instead of false misrepresentation. Using Section 27
of the Insurance Code, it argued that the concealment of age whether intentional or not entitled
the company to rescind the insurance contract. It further argued that it did not rely solely on the
Health Statement Form of Alvarez but also with the representations during the background check
conducted by Union Bank stating that Alvarez was only 55 years old at the time of application.
Being a contract uberrima fides, it has every right to rely on the good faith of Alvarez in its
dealing with him.
Union Bank argued that the real estate mortgage is not affected by the status of Group
Mortgage Redemption Insurance because they are different contracts. Any concealment should
not result to the invalidation of the foreclosure it made to the property of Alvarez.

ISSUE: Whether or not Insular Life can validly rescind the contract because of concealment.

RULING:
Insular Life cannot validly rescind the contract because of concealment. It is liable under
Group Mortgage Redemption Insurance.

Insular Life correctly pointed out the meaning of Section 27 of Insurance Code which
provides that: A concealment whether intentional or unintentional entitles the injured party to
rescind a contract of insurance. It correctly pointed out that for the purpose of rescission under
Section 27 of the Insurance Code, distinction between intentional and unintentional
concealments is not necessary.

However, Insular Assurance erroneously pleaded Section 27 to defend its case. The
applicable provision in this case is Section 44 which provides that “A representation is to be
deemed false when facts fail to correspond with its assertions or stipulations”. According to the
Supreme Court, if indeed Alvarez misdeclared his age such that his assertion fails to correspond
with his factual age, he made false representation, not concealment. Section 45 is the counterpart
provision of Section 27. Section 45 provides that “If a representation is false in a material point,
whether affirmative or promissory, the injured party is entitled to rescind the contract from the
time when the representation becomes false.” Unlike Section 27, Section 45 remains subject to
the basic precept of fraud, which is to be proved by clear and convincing evidence, which was
not established in the case.

According to the Supreme Court, when the insured makes a representation, it is


incumbent on them to assure themselves that a representation on the material fact is not false;
and if it is false, that is not fraudulent misrepresentation of material fact. This returns the burden
to insurance companies, which in general, have more resources than the insured to check the
veracity of insured’s beliefs as to a statement of fact. “Consciousness in defraudation is
imperative and it is for the insurer to show this. Insular failed to establish such fraud as it failed
to provide clear and convincing evidence.
Quoting from the decision: the most basic document that Alvarez accomplished in
relation to Insular Life must have been the application form. Strangely, Insular Life failed to
adduce even this document- a piece of evidence that was not only commonsensical, but also one
which has always been in its possession and disposal. Insular claimed that it did not rely solely
on Alvarez’s Health Statement Form but also on his Background Checking Report. Reliance on
this report is problematic. It was not prepared by Alvarez. Rather, it was prepared by Union
Bank employee following the conduct of credit investigation. Hence, Insular Life is liable under
Group Mortgage Redemption Insurance.

The Court added that Alvarez entered into the Group Mortgage Redemption Insurance
entirely upon Union Bank’s prodding. Bank clients are generally unaware of insurance policies
such as a mortgage redemption insurance unless brought to their knowledge by a bank. The
processing of a mortgage redemption insurance was within Union Bank’s regular course of
business. It knew the import of truthfully and carefully accomplished applications. To facilitate
the principal contract of the loan and its accessory obligations such as the real estate mortgage
and the mortgage redemption insurance, Union Bank completed credit appraisals and
background checks. Union had the necessary materials sufficient to inform itself of Alvarez’s
personal circumstances.

 Musngi vs. West Coast Life Insurance, 61 Phil. 864

FACTS:
Arsenio Garcia was insured by West Coast Life Insurance Company twice in July 1931
and October 1931 respectively. These two policies were valid and binding at the time of the
insured’s death on December 1932. In both applications, the insured had to answer – “What
physician have you consulted or been treated by and for what illness?” He answered in the said
applications “none” and “no.” However, the defendant company discovered that the truth was
that before said incidents of answering, signing and issuance of contract, Arsenio Garcia had
been treated in the General Hospital for different ailments and for several occasions.

The petitioners herein are beneficiaries in the said policies demanding for payment but
the defendant company refused to do so.

ISSUE/S:
Whether or not the answers given by the insured in his applications are false and if they
were the cause or one of the causes which induced the defendant company to issue the policies

RULING:
Yes. The answers given were false and were one of the causes which induced the
defendant company to issue the policies.

The insured knew that he had suffered from a number of ailments, including
incipient pulmonary tuberculosis, before subscribing the applications, yet he concealed them and
omitted the hospital where he was confined as well as the name of the lady physician who treated
him. This concealment and the false statements constituted fraud, is likewise clear, because the
defendant by reason thereof accepted the risk which it would otherwise have flatly refused. The
two answers being one of the considerations of the policies, and it appearing that they are false
and fraudulent, it is evident that the insurance contracts were null and void and did not give rise
to any right to recover their value or amount.

Furthermore, one ground for the rescission of a contract of insurance under the
Insurance Act is a 'concealment', which in section 25 is defined as 'A neglect to communicate
that which a party knows and ought to communicate'. The basis of the rule vitiating the contract
in cases of concealment is that it misleads or deceives the insurer into accepting the risk, or
accepting it at the rate of premium agreed upon. The principal question, therefore, must be, Was
the assurer misled or deceived into entering a contract obligation or in fixing the premium of
insurance by a withholding of material information or facts within the assured's knowledge or
presumed knowledge? The assurer in assuming a risk is entitled to know every material fact of
which the assured has exclusive or peculiar knowledge, as well as all material facts which
directly tend to increase the hazard or risk which are known by the assured, or which ought to be
or are presumed to be known by him. And a concealment of such facts vitiates the policy. lf the
assured has exclusive knowledge of material facts, he should fully and fairly disclose the same,
whether he believes them material or not.

In an action on a life insurance policy where the evidence conclusively shows that
the answers to questions concerning diseases were untrue, the truth or falsity of the answers
become the determining factor. If the policy was procured by fraudulent representations, the
contract of insurance apparently set forth therein was never legally existent.

Applying the aforementioned rules of law in the case at bar, there was a
concealment of the material fact which vitiates the contract. Hence, the petitioner beneficiaries
are not entitled to any right of recovery of any amount from defendant company.

 Argente vs. West Coast Life Insurance, 51 Phil. 725

F A C T S:

On 9 Feb. 1925, Sps. Bernardo and Vicenta Argente signed an application for joint insurance
with West Coast Life Insurance Co. (West for brevity) in the sum of P2,000. On 9 May 1925,
the spouses submitted to West an amended application for insurance, increasing the amount to
P15,000. On 18 Nov. 1925, however, Vicenta died of cerebral apoplexy, causing Bernard to file
a claim with West.

Following an investigation, West refused to grant the claim because the answers given by the
spouses in their medical examination were untrue. It was stated that, among others, Vicenta only
drinks beer in small quantities and she has no ailment of the brain or nervous system.
Nonetheless, it was found out that sometime in 1924, Vicenta was taken by a patrolman at the
request of Bernardo to San Lazaro Hospital due to alcoholism, manic-depressive psychosis, and
was finally diagnosed of psycho-neurosis.

I S S U E:

Whether Sps. Ocampo were guilty of concealment and thereby misled West Cost Life Insurance
to accept the risk.

H E L D:

Yes, the Court gave credence to the trial court’s finding that the representation made by Vicenta
in her application for life insurance were false with respect to her state of health and that she
knew and was aware that the representations so made by her were false.

Concealment, being one of the grounds for recission under the Insurance Code, is defined by
Sec. 25 as a neglect to communicate that which a party knows and ought to communicate.
Concealment exists where the assured has knowledge of a fact material to the risk, and honesty,
good faith, and fair dealing requires that he should communicate it to the insurer, but he
designedly and intentionally withheld the same. The basis of the rule vitiating the contract in
cases of concealment is that it misleads or deceives the insurer into accepting the risk, or
accepting it at the rate of premium agreed upon. The insurer is misled into a belief that the
circumstances withheld does not exists, and he is thereby induced to estimate the risk upon a
false basis that does not exist.

It does not seem to be necessary that the suppression of the truth should have been willful. If it
were but an inadvertent omission, yet if it were material to the risk and such as the plaintiff
should have known to be so, it would render the policy void. But it is held that is untrue or false
answers are given in response to inquiries and they relate to material facts, the policy is avoided
without regard to the knowledge or fraud of assured. The policy will be avoided by a
nondisclosure irrespective of the fact whether the assured knew of such ailment or not.

 Grepalife vs. CA, L-31845, April 30, 1979

FACTS:
Private respondent Ngo Hing filed a life insurance application with Great Pacific Life
Assurance (Pacific life) for a 20-year endowment policy in the amount of 50,000 on the life of
his 1 year old daughter Helen Go. Lapu-lapu Mondragon, the branch manager of Pacific life in
Cebu type-wrote the data on the application form which was signed by Ngo Hing. Not mentioned
in the application, the fact that Helen Go was a special child. Thereafter, Ngo Hing paid the
annual premium amounting to P1,077.00 but he retained the amount of P1317.00 as his
commission as a duly authorized agent of Pacific Life. Upon the payment of the premium, a
binding deposit receipt was issued to Ngo Hing. Also, at the back of the application form,
Mondragon wrote his strong recommendation for the application of Ngo Hing. Later, Mondragon
received a letter from Pacific life about its disapproval of the application because the life
insurance application of a 20-year endowment plan is not available for minors below 7 years of
age. In the same letter, Pacific life recommended that the Juvenile Triple Action Plan be instead
offered to Ngo Hing. The non-acceptance of the offer was not communicated by Mondragon to
Ngo Hing. Instead, he wrote back to Pacific Life pointing out that since 1954, the customers
especially the Chinese were asking for such coverage.

Eventually, Helen Go died of influenza with complications of bronchopneumonia. Consequently,


Ngo Hing sought the payment of the proceeds of the insurance, but having failed in his efforts,
he filed an action for recovery of the same before the CFI Cebu. CFI Cebu granted the said claim
of Ngo Hing. Hence, this case.

ISSUE/S:

1) Whether the binding deposit receipt constituted as a temporary contract of life


insurance.

2.) Whether Ngo Hing concealed the state of health and physical condition of Helen Go,
which rendered the binding deposit receipt as void.

RULING:
1) No, the Supreme Court ruled that, the binding deposit receipt in question is merely an
acknowledgment, on behalf of the company, that the latter's branch office had received from the
applicant the insurance premium and had accepted the application subject for processing by the
insurance company; and that the latter will either approve or reject the same on the basis of
whether or not the applicant is "insurable on standard rates." Since petitioner Pacific Life
disapproved the insurance application of respondent Ngo Hing, the binding deposit receipt in
question had never become in force at any time.

Upon this premise, the binding deposit receipt is, manifestly, merely conditional and does not
insure outright. As held by this Court, where an agreement is made between the applicant and the
agent, no liability shall attach until the principal approves the risk and a receipt is given by the
agent. The acceptance is merely conditional and is subordinated to the act of the company in
approving or rejecting the application.

It bears repeating that through the intra-company communication of April 30, 1957, Pacific Life
disapproved the insurance application in question on the ground that it is not offering the twenty-
year endowment insurance policy to children less than seven years of age. What it offered
instead is another plan known as the Juvenile Triple Action, which private respondent failed to
accept. In the absence of a meeting of the minds between petitioner Pacific Life and private
respondent Ngo Hing over the 20-year endowment life insurance in the amount of P50,000.00 in
favor of the latter's one-year old daughter, and with the non-compliance of the above-quoted
conditions stated in the disputed binding deposit receipt, there could have been no insurance
contract duly perfected between them. Accordingly, the deposit paid by private respondent shall
have to be refunded by Pacific Life.

A contract of insurance, like other contracts, must be assented to by both parties either in person
or by their agents ... The contract, to be binding from the date of the application, must have been
a completed contract, one that leaves nothing to be done, nothing to be completed, nothing to be
passed upon, or determined, before it shall take effect. There can be no contract of insurance
unless the minds of the parties have met in agreement.

2) Yes, the Supreme Court is of the firm belief that private respondent had deliberately concealed
the state of health and physical condition of his daughter Helen Go. When private respondent
supplied the required essential data for the insurance application form, he was fully aware that
his one-year old daughter is typically a mongoloid child. Such a congenital physical defect could
never be ensconced nor disguised. Nonetheless, private respondent, in apparent bad faith,
withheld the fact material to the risk to be assumed by the insurance company. As an insurance
agent of Pacific Life, he ought to know, as he surely must have known his duty and
responsibility to such a material fact. Had he diamond said significant fact in the insurance
application form Pacific Life would have verified the same and would have had no choice but to
disapprove the application outright.
The contract of insurance is one of perfect good faith uberrima fides meaning good faith,
absolute and perfect candor or openness and honesty; the absence of any concealment or
demotion, however slight, not for the alone but equally so for the insurer. Concealment is a
neglect to communicate that which a party knows and ought to communicate. Whether
intentional or unintentional the concealment entitles the insurer to rescind the contract of
insurance.

b. representation
1. concept
2. kinds
3. test of materiality
4. effect of alteration or withdrawal
5. time to which representation refers
6. effect when representation is obtained from third persons
7. when presumed false; effect of falsity

 Ma. Lourdes S. Florendo vs. Philam Plans, GR 186983, February 22, 2012

FACTS:

Manuel Florendo filed an application for comprehensive pension plan with respondent
Philam Plans. Manuel signed the application and left to Perla the task of supplying the
information needed in the application. Respondent Ma. Celeste Abcede, Perla’s daughter, signed
the application as sales counselor. The comprehensive pension plan also provided life insurance
coverage to Florendo, with a condition that if the plan holder died before the maturity of the
plan, his beneficiary was to instead receive the proceeds of the life insurance, equivalent to the
pre-need price. Further, the life insurance was to take care of any unpaid premium until the
pension plan matured, entitling the beneficiary to the maturity value of the pension plan.

Philam Plans issued Pension Plan Agreement to Manuel, with Ma. Lourdes S. Florendo,
his wife, as beneficiary. Manuel paid his quarterly premiums but eleven months later, Manuel
died of blood poisoning. Lourdes then filed a claim with Philam Plans for the payment of the
benefits under her husband’s plan but Philam Plans declined her claim because it was found that
Manuel was on maintenance medicine for his heart and had an implanted pacemaker. Further, he
suffered from diabetes mellitus and was taking insulin. Lourdes filed the present action against
the pension plan company before the RTC-Quezon City who ruled in favor of Ma. Lourdes.
However, the Court of Appeals then reversed the RTC decision, holding that Manuel to disclose
to Philam Plans conditions affecting the risk of which he was aware or material facts that he
knew or ought to know.

ISSUES:
1. Whether Manuel is guilty of concealing his illness when he kept blank and did not
answer questions in his pension plan application;
2. Whether Manuel was bound by the failure of Perla and Ma. Celeste to declare the
condition of Manuel’s health;
3. Whether Philam Plans’ approval of Manuel’s pension plan application and acceptance of
his premium payments precluded it from denying Lourdes’ claim.

RULING:

1. YES.

When Manuel signed the application, he adopted as his own the written representations
and declarations embodied in it. It is clear from these representations that he concealed his
chronic heart ailment and diabetes from Philam Plans. The pertinent portion of his
representations and declarations read as follows: (c) I have never been treated for heart
condition, high blood pressure, cancer, diabetes, lung, kidney or stomach disorder or any other
physical impairment in the last five years. (d) I am in good health and physical condition.
Manuel signed the application without filling in the details regarding his continuing treatments
for heart condition and diabetes. The assumption is that he has never been treated for the said
illnesses in the last five years preceding his application.

Lourdes insists that Perla, the soliciting agent, knew that Manuel had a pacemaker before
he signed up for the pension plan.23 But by its tenor, the responsibility for preparing the
application belonged to Manuel. Nothing in it implies that someone else may provide the
information that Philam Plans needed. Manuel cannot sign the application and disown the
responsibility for having it filled up. If he furnished Perla the needed information and delegated
to her the filling up of the application, then she acted on his instruction, not on Philam Plans’
instruction.

Manuel still had his pacemaker when he applied for a pension plan and it is an admission
that he remained under treatment for irregular heartbeat within five years preceding that
application. Manuel had been taking medicine when he submitted his pension plan application.
These clearly fell within the five-year period. It is not claimed that Perla was aware of his two
other afflictions that needed medical treatments. Pursuant to Section 27 of IC, Manuel’s
concealment entitles Philam Plans to rescind its contract of insurance with him.

2. YES.

Manuel, in signing the pension plan application, he certified that he wrote all the
information stated in it or had someone do it under his direction. Assuming that it was Perla who
filled up the application form, Manuel is still bound by what it contains since he certified that he
authorized her action. Philam Plans had every right to act on the faith of that certification.
Manuel was made aware when he signed the pension plan application that, in granting the same,
Philam Plans and Philam Life were acting on the truth of the representations contained in that
application.

Manuel, a civil engineer and manager of a construction company, could be expected to


know that one must read every document, especially if it creates rights and obligations affecting
him, before signing the same. It could reasonably be expected that he would not trifle with
something that would provide additional financial security to him and to his wife in his twilight
years.

3. NO.

The comprehensive pension plan that Philam Plans issued contains a one-year incontestability
period. The incontestability clause precludes the insurer from disowning liability under the
policy it issued on the ground of concealment or misrepresentation regarding the health of the
insured after a year of its issuance.

Since Manuel died in the eleventh month following the issuance of his plan, the one year
incontestability period has not yet set in.

 Emilio Tan vs. CA, GR 48049, June 29, 1989

FACTS:

Tan Lee Siong, father of the petitioners, applied for life insurance in the amount of
P80,000.00 with respondent company Philippine American Life Insurance Company. He stated
in the application form that he has no health issues whatsoever and so in November 1973 he was
issued a life insurance policy in the amount of P80,000.00 and it was approved. He listed his
sons as beneficiaries (Emilio Tan et al). On 26 April 1975, Tan Lee Siong died due to hepatoma.
His sons filed an insurance claim but PHILAMLIFE denied the same as it alleged that Tan Lee
Siong concealed the fact that he was hypertensive, diabetic, and was suffering from hepatoma at
the time of his application for the insurance. The premiums paid on the policy were thwn
refunded. The petitioners contend that the respondent company no longer had the right to rescind
the contract of insurance as rescission must allegedly be done during the lifetime of the insured
within two years and prior to the commencement of action.

ISSUE/S:

Whether or not the insurance company has the right to rescind the contract of insurance
RULING:

Yes. The Insurance company has the right to rescind the contract of insurance despite the
“incontestability clause”. The so-called “incontestability clause” precludes the insurer from
raising the defenses of false representations or concealment of material facts insofar as health
and previous diseases are concerned if the insurance has been in force for at least two years
during the insured’s lifetime. The phrase “during the lifetime” found in Section 48 of the
Insurance Law simply means that the policy is no longer considered in force after the insured has
died. The key phrase in the second paragraph of Section 48 is “for a period of two years”. In the
present case, the policy was issued on November 6, 1973 and the insured died on April 26, 1975.
Hence, the policy was in force for a period of only one year and five months. Considering that
the insured died before the two-year period has lapsed, respondent company is not, therefore,
barred from proving that the policy is void ab initio by reason of the insured’s fraudulent
concealment or misrepresentation. Furthermore, respondent company rescinded the contract of
insurance and refunded the premiums paid on November 11, 1975, previous to the
commencement of this action on November 27, 1975.

 Manila Bankers Life vs. Cresencia P. Aban, GR 175666, July 29, 2013

F A C T S:

On 3 July 1993, Delia Sotero (Sotero for brevity) took out a life insurance policy from Manila
Bankers Life Insurance Corp. (petitioner for brevity) naming Cresencia Aban (Aban for brevity).
On 10 April 1996, Sotero died, prompting Aban to file a claim for the insurance proceeds.
However, after conducting investigation, respondent denied such claim for reasons of fraud and
concealment, arguing that Aban was the one who filed Sotero’s claim on 3 July 1993 and
designated herself as the beneficiary.

Petitioner then filed a case for recission and/or annulment of the policy. Aban countered with a
motion to dismissed, which the trial court granted. The RTC ruled that it was Sotero, not Aba,
who procured the insurance. Also, under Sec. 48 of the Insurance Code, petitioner had only 2
years from the effectivity of the insurance policy to question the same; petitioner is now barred
from contesting the policy. The CA affirmed RTC’s decision.

I S S U E:

Whether petitioner Manila Bankers Life Insurance is barred from denying the insurance claims
based on fraud or concealment.
H E L D:

Yes, the petitioner is barred from denying the insurance by reason of the “incontestability
clause” under Sec. 48[1] of the Insurance Code. The reason for such clause is that insurers who
recklessly and indiscriminately solicit and obtain business must be penalized, for such reckless
and lack of indiscrimination ultimately work to the detriment of bona fide takers of insurance
and public in general. It compels the insurers to solicit business from or provide insurance
coverage only to legitimate and bona fide clients, by requiring them to thoroughly investigate
those they insure within 2 years from effectivity of the policy and while the insured is still alive.
The law assumes that they will do just that and not sit on their laurels.

Sec. 48. “Whenever a right to rescind a contract of insurance is given to the insurer by
any provision of this chapter, such right must be exercised previous to the commencement of an
action on the contract. After a policy of life insurance made payable on the death of the insured
shall have been in force during the lifetime of the insured for a period of 2 years from the date of
its issue or of its last reinstatement, the insurer cannot prove that the policy is void ab initio or is
rescindable by reason of the fraudulent concealment or misrepresentation of the insured or his
agent.”

Sec. 48 regulates both the actions of the insurers and prospective takers of life insurance. It
gives insurers enough time to inquire whether the policy was obtained by fraud, concealment, or
misrepresentation; o the other hand, it forewarns scheming individuals that their attempts at
insurance fraud would be timely uncovered – thus deterring them from venturing into such
nefarious enterprise. At the same time, legitimate policy holders are protected from unwarranted
denial of their claims or delay in the collection of insurance proceeds occasioned by allegations
of fraud, concealment, or misrepresentation by insurers, claims which may no longer be set up
after the 2-year period expires.

Life insurance policies that pass the statutory 2-year period are essentially treated as legitimate
and beyond question, and the individual who wield them are made secure by the thought that
they will be paid promptly upon claim.

In the instant case, instead of investigating at the first instance, petitioner appears to have turned
a blind eye and opted instead to continue collecting the premiums on the policy. For nearly 3
years, petitioner collected the premiums and devoted the same to its own profit. Insurers cannot
be allowed to collect premiums on insurance policies, use these amounts collected and invest the
same through the years, generating profits and returns therefrom for their own benefit, and
thereafter conveniently deny insurance claims by questioning the integrity of such policy.
The business of insurance is a highly regulated commercial activity, and is imbued with public
interest. An insurance contract is a contract of adhesion which must be construed liberally in
favor of the insured and strictly against the insurer in order to safeguard the former’s interest.

 Insular Life vs. Heirs of Alvarez, GR 207526 and 210156, October 3, 2018

FACTS:

Jose H. Alvarez and his wife owned a residential lot in Caloocan. In 1997, Union Bank
approved the housing loan of Alvarez in the amount of 648,000. A promissory note and real
estate secured the loan. Jose Alvarez was included among the mortgagors in the list of qualified
debtors covered by the Group Mortgage Redemption Insurance that Union Bank had with Insular
Life. A Mortgage Redemption Insurance was taken in the life of Alvarez. Jose Alvarez died in
April 17, 1998.
Union Bank filed with Insular Life a death claim under Alvarez’s name pursuant to the
Group Mortgage Redemption Insurance. Insular Life denied the claim because Alvarez did not
qualify for the coverage. He was supposedly more than 60 years when he applied for the loan.
The monthly amortizations of Alvarez stood unpaid. The lot was foreclosed and it was sold in
public auction having Union Bank as the highest bidder.
In 2001, the heirs of Alvarez filed a complaint for Declaration of the Nullity of Contract
and Damages against Union Bank, Alfonso Miranda (supposedly benefitted from the loan), and
the insurer identified only as John Doe. The heirs denied having any knowledge of the loan
obtained by Alvarez. The complaint was amended and became specific performance including
demand against Insular Life to fulfill its obligation as insurer under Group Mortgage Redemption
Insurance.
Union Bank argued against the ignorance raised by the heirs because of the Special
Power of Attorney executed by Adelina in favor of her husband.
The RTC ruled in favor of the heirs of Alvarez. It ruled that Alvarez had no fraudulent
intent when he gave Union Bank information about his age and date of birth. According to RTC,
Union Bank negotiated the Group Mortgage Insurance with Insular Life and ordinary customers
will not know about insurance policies unless it is brought to their knowledge by the bank. Also,
Union Bank had been in possession of the materials sufficient to inform itself of Alvarez
personal circumstances.
Insular Life argued for concealment instead of false misrepresentation. Using Section 27
of the Insurance Code, it argued that the concealment of age whether intentional or not entitled
the company to rescind the insurance contract. It further argued that it did not rely solely on the
Health Statement Form of Alvarez but also with the representations during the background check
conducted by Union Bank stating that Alvarez was only 55 years old at the time of application.
Being a contract uberrima fides, it has every right to rely on the good faith of Alvarez in its
dealing with him.
Union Bank argued that the real estate mortgage is not affected by the status of Group
Mortgage Redemption Insurance because they are different contracts. Any concealment should
not result to the invalidation of the foreclosure it made to the property of Alvarez.

ISSUE: Whether or not Insular Life can validly rescind the contract because of concealment.

RULING:
Insular Life cannot validly rescind the contract because of concealment. It is liable under
Group Mortgage Redemption Insurance.

Insular Life correctly pointed out the meaning of Section 27 of Insurance Code which
provides that: A concealment whether intentional or unintentional entitles the injured party to
rescind a contract of insurance. It correctly pointed out that for the purpose of rescission under
Section 27 of the Insurance Code, distinction between intentional and unintentional
concealments is not necessary.

However, Insular Assurance erroneously pleaded Section 27 to defend its case. The
applicable provision in this case is Section 44 which provides that “A representation is to be
deemed false when facts fail to correspond with its assertions or stipulations”. According to the
Supreme Court, if indeed Alvarez misdeclared his age such that his assertion fails to correspond
with his factual age, he made false representation, not concealment. Section 45 is the counterpart
provision of Section 27. Section 45 provides that “If a representation is false in a material point,
whether affirmative or promissory, the injured party is entitled to rescind the contract from the
time when the representation becomes false.” Unlike Section 27, Section 45 remains subject to
the basic precept of fraud, which is to be proved by clear and convincing evidence, which was
not established in the case.

According to the Supreme Court, when the insured makes a representation, it is


incumbent on them to assure themselves that a representation on the material fact is not false;
and if it is false, that is not fraudulent misrepresentation of material fact. This returns the burden
to insurance companies, which in general, have more resources than the insured to check the
veracity of insured’s beliefs as to a statement of fact. “Consciousness in defraudation is
imperative and it is for the insurer to show this. Insular failed to establish such fraud as it failed
to provide clear and convincing evidence.

Quoting from the decision: the most basic document that Alvarez accomplished in
relation to Insular Life must have been the application form. Strangely, Insular Life failed to
adduce even this document- a piece of evidence that was not only commonsensical, but also one
which has always been in its possession and disposal. Insular claimed that it did not rely solely
on Alvarez’s Health Statement Form but also on his Background Checking Report. Reliance on
this report is problematic. It was not prepared by Alvarez. Rather, it was prepared by Union
Bank employee following the conduct of credit investigation. Hence, Insular Life is liable under
Group Mortgage Redemption Insurance.

The Court added that Alvarez entered into the Group Mortgage Redemption Insurance
entirely upon Union Bank’s prodding. Bank clients are generally unaware of insurance policies
such as a mortgage redemption insurance unless brought to their knowledge by a bank. The
processing of a mortgage redemption insurance was within Union Bank’s regular course of
business. It knew the import of truthfully and carefully accomplished applications. To facilitate
the principal contract of the loan and its accessory obligations such as the real estate mortgage
and the mortgage redemption insurance, Union Bank completed credit appraisals and
background checks. Union had the necessary materials sufficient to inform itself of Alvarez’s
personal circumstances.

c. remedies available in case of concealment or false representation


1. rescission by the insurer
2. when life insurance policy becomes incontestable
a. requisites for incontestability
b. theory and object of incontestability
c. defenses not barred by incontestability
d. warranties
1. concept; distinguished from representation
2. kinds of warranties (express, implied, affirmative, promissory)
3. time to which warranty refers
4. effect of breach

Great Pacific Life Assurance vs. CA, 316 SCRA 678


Sun Life Assurance Co. of Canada vs. CA, G.R. No. 105135, June 22, 1995
Philamcare Health Systems Inc. vs. CA, 379 SCRA 356
Vda de Canilang vs. CA, G.R. No. 92492, June 17, 1993
Tan vs. CA, June 29, 1989
Prudential Guarantee vs. Trans-Asia Shipping Lines, GR 151890, June 20, 2005

Vous aimerez peut-être aussi